HMX Physiology

Pataasin ang iyong marka sa homework at exams ngayon gamit ang Quizwiz!

What forces contribute to the water balance between the intracellular space and the interstitial space? oncotic and hydrostatic osmotic only oncotic only hydrostatic only

oncotic only Explanation: Movement of fluid between the intracellular and interstitial compartments is governed by osmotic forces. Hydrostatic pressures inside and outside of cells are essentially the same, given that cell walls are highly flexible structures.

A typical healthy adult might have a cardiac output of 5 L/min, a mean arterial pressure (MAP) of 95 mmHg, and a central venous pressure (CVP) of 5 mmHg. (Central venous pressure is also sometimes just referred to as "venous pressure"). Using these parameters, calculate systemic vascular resistance (SVR) of this example adult in mmHg⋅min / mL (note that this is in mL, not liters). Express your answer as a number only; do not write the units at the end and do not round.

.018 or 0.018 Explanation First, we need to rearrange the following equation: P=Q*SVR Divide both sides by Q: We know that P= MAP-CVP P= 95 mmHg - 5 mmHg = 90 mmHg Therefore: SVR = 90 mmHg / (5 L / min) = 90 mm Hg / (5000 mL / min) = .018 mmHg⋅min / mL

A patient weighs 75kg and ingests 45g of a product that equally distributes into all 3 compartments of the body (15g in each). Assume one third of the extracellular space is intravascular and assume % total body water as discussed in the Concept Videos. After distribution, what will be the concentration of the product (assume 1kg/L as density of water) in each compartment? 1g/L intracellular; 2g/L interstitial; 3g/L intravascular 3g/L intracellular; 2g/L interstitial; 1g/L intravascular 15g/L i

0.5 g/L intracellular; 1.5g/L interstitial; 3g/L intravascular Explanation: Assuming 60% water→45kg(L) total body water →30L IC and 15L EC → 10L IS and 5L IV → 15g/30L; IC 15g/10L IS; 15g/5L IV where IC = intracellular; IV = intravascular; EC = extracellular; IS = interstitial.

The relationship between alveolar ventilation (mL/min), carbon dioxide production (mL/min), and partial pressure of carbon dioxide in the blood (mm Hg) is written below. If the alveolar ventilation increases from 500 mL/min to 1,000 mL/min and carbon dioxide production remains constant, what would be the new value for the partial pressure of carbon dioxide in the blood (assuming a starting value of P)? *You may note that the units do not cancel because there is a constant missing that is necessa

0.5P Explanation If elimination of carbon dioxide (the alveolar ventilation) is increased by a factor of 2, the new steady state for partial pressure of carbon dioxide in the blood must decrease by a factor of 2, meaning the new partial pressure is 0.5P.

Oxygen delivery is dependent on cardiac output (volume of blood pumped per minute in mL/min) and the oxygen content of the blood. The average person has an oxygen delivery between 600 and 1400 mL/min. Lance, an elite cyclist, has a resting heart rate of 30 beats/min and a stroke volume of 200 mL/beat. Assuming he has a normal arterial oxygen content of 20 mL/dL, what is his O2 delivery at rest? 1,200 mL/min 3,600 mL/min 36,000 mL/min 120,000 mL/min

1,200 mL/min Explanation Oxygen delivery (DO2) is the product of cardiac output (Q) and arterial oxygen content, where Q is the product of heart rate and stroke volume. For Lance at rest: DO2 = (30 beats/min * 200 mL/beat) * (20 mL/dL * 1 dL/100 mL) = 1200 mL/min. If you find it surprising that this value falls within the normal range, consider that at maximal exertion, Lance's heart rate can reach 200 beats/min, which leads to an oxygen delivery of 8,000 mL/min if all other factors remain constant.

Each of the scenarios below describes a change in the pressure inside of (Pin) and outside of (Pout) a balloon. Select the correct change in volume for each scenario. 1. Pin increases, Pout decreases 2. Pin increases, Pout constant 3. Pin increases, Pout increases 4. Pin constant, Pout increases 5. Pin constant, Pout decreases 6. Pin decreases, Pout increases 7. Pin decreases, Pout constant 8. Pin decreases, Pout decreases

1. balloon volume increases 2. balloon volume increases 3. need more information 4. balloon volume decreases 5. balloon volume increases 6. balloon volume decreases 7. balloon volume decreases 8. need more information Explanation Transmural pressure (PTM = Pin - Pout) dictates the final volume of a flexible structure. When the transmural pressure increases, the volume increases, and vice-versa. In examples where the difference between Pin - Pout increases, the volume will increase. The opposite is also true: for examples in which the difference between Pin - Pout decreases, the volume will decrease. Lastly, when Pin and Pout change in the same direction, you must know the magnitude of the change of each to determine whether transmural pressure increases or decreases. The correct answers are indicated above.

We've been mainly focused so far on scuba divers breathing pressurized gas. Now let's consider a diver holding his or her breath. Assume the lungs are like isolated balloons for this example (ignore the stiffness of the enclosing chest wall). If the lung volume on the surface after the breath is V0, what is the lung volume at 66 ft (3.00 atm of water pressure) relative to V0? V0 1/2 V0 1/3 V0 3 x V0

1/3 V0 Explanation: Boyle's Law describes the inverse relationship between pressure and volume for a fixed amount of gas at constant temperature. In this example, the surrounding pressure has increased threefold (1.00 atm to 3.00 atm) and hence the pressure in the lungs also approximately triples. Given that the product of pressure and volume (P0 x V0 on the surface) stays constant, lung volume must decrease to one third of its original value (3.00 P0 x 1/3 V0 = P0 x V0). If this is unclear, review the diagram and make sure to select the information icon in the lower right to see a more in-depth explanation.

A healthy 21 year old man is mountain climbing in Nepal and is at an altitude of approximately 3,000 meters. Barometric pressure is 530 mm Hg. He is feeling short of breath and the climbing is becoming more difficult. The partial pressure of oxygen in the air he is breathing is approximately: 150 mm Hg 220 mm Hg 110 mm Hg 35 mm Hg

110 mm Hg Explanation: Atmospheric pressure at sea level is 760 mmHg or one atmosphere (atm). As we ascend, the pressure decreases (~0.012 atm or 9mmHg per 100 meters) and thus the partial pressures of atmospheric gases drop (even though the composition or percentages remain the same). Partial pressure of a gas is calculated as the concentration (percentage) of that gas multiplied by the atmospheric pressure (Pgas = Patm (Fgas) ). Air is composed of 78% nitrogen (N2) and 21% oxygen (O2) and 1% other gases (these values are sometimes simplified to 79% N2 and 21% O2 for calculations). Thus the PO2 or partial pressure of oxygen is 0.21 X 530 = 111.3 or about 110mmHg.

Assume the vascular compartment comprises 1/3 of the volume of the extracellular compartment. If you drink a liter of water, how much will end up in the blood vessels? Assume proportions shown in the Body Fluid Compartments video. 111 ml 333 ml 167 ml 666 ml

111 ml Explanation: A simplified approximation, useful as a mnemonic, is that extracellular fluid is 1/3 of total body water, and intravascular fluid is 1/3 of the extracellular fluid. Based on those proportions, intravascular fluid is roughly 1/9 of total body water. Ingested fluid moves across vessel walls and cell walls, eventually equilibrating such that 1/9 of a liter, or 111 ml, ends up in the intravascular space. It may surprise you that so little of the water you drink ends up in your blood.

Suppose you have a container of liquid that has equilibrated with the air above its surface, such the PO2 in the liquid is approximately 160 mm Hg. You then very rapidly exchange the gas above the liquid for a new gas that has a total pressure of 1520 mm Hg (twice atmospheric pressure) and a PO2 of 160 mm Hg. What is the PO2 in the liquid immediately after the switch and one hour after the switch? 160 mm Hg immediately after; 320 mm Hg one hour later 160 mm Hg immediately after; 160 mm Hg one h

160 mm Hg immediately after; 160 mm Hg one hour later Explanation: The PO2 in the gas determines the PO2 in the liquid. Equilibration is not instantaneous (O2 molecules have to enter and leave the liquid), but that doesn't come into play with the PO2 in this case since it is unchanged. Immediately after the switch, the PO2 of the liquid is still 160 mm Hg. An hour later, the PO2 of the gas above the liquid is still 160 mm Hg (the new gas has a different fraction of oxygen than atmospheric air but the same PO2) and thus the PO2 of the liquid is still the same. It is the partial pressure of the specific gas (O2 in this case) that matters for the PO2 of the liquid, not the total pressure of the new gas.

If the production of carbon dioxide in the body doubles, what must be the new value for alveolar ventilation (assuming a starting value V) in order to maintain a constant partial pressure of carbon dioxide in blood? The relevant equation (shown without a proportionality constant) is: Alveolar ventilation = (carbon dioxide production) / (partial pressure of carbon dioxide in blood) 0.5V 1.0V 1.5V 2.0V

2.0V Explanation According to the equation, if production of carbon dioxide doubles, elimination (alveolar ventilation) must also double to maintain a constant steady state concentration of carbon dioxide in the blood. This elimination process is perturbed in many patients with lung disease, resulting in chronically elevated levels of carbon dioxide in the blood. Note that in the videos and in previous questions, the attainment of a new steady state after a system enters dynamic conditions was discussed. Here, we are asking what must happen to prevent the body from going into a dynamic state with respect to carbon dioxide levels in the first place.

The kidney filters potentially toxic substances in the blood, and thus "clears" the blood of those substances. This clearance function is dependent upon and proportional to the diffusion gradient of the substance across filtering capillaries, i.e. if the concentration of the substance is doubled, twice as much will be cleared from each ml of blood that is filtered. Suppose that the body produces a constant amount of a substance X per unit of time. The kidneys eliminate substance X at a rate dire

2.0[X]0 Explanation In this simplified dynamic system, the amount of a substance cleared from a given volume of blood is proportional to the concentration of that substance. If the clearance halves, the concentration must double for her to eliminate the same amount of substance X. Thus, this will be the new steady state concentration of X.

At 66 feet of depth, the scuba diver is breathing air at 3.00 atm of pressure. What is the inspired fraction of oxygen at that point? Try to answer without re-visiting the diagram. 63% 42% 21% 7%

21% Explanation: Although the total inspired pressure changes and the PO2 in the inspired air rises proportionally, the fraction of inhaled oxygen does not change. Most recreational scuba dives are done using compressed air. If this is unclear, review the diagram and make sure to select the information icon in the lower right to see a more in-depth explanation.

A 67-year-old man arrives in the emergency department complaining of fatigue, confusion, headache, and loss of appetite. When you take his medical history, he reveals that he has advanced renal disease and recently missed a dialysis appointment. You order a basic metabolic panel and an arterial blood gas. These tests reveal the following pertinent values: pH = 7.22 PCO2 = 23 mEq/L PO2 = 98 mEq/L HCO3- = 10 mEq/L Na+ = 140 mEq/L K+ = 5.0 mEq/L Cl- = 110 mEq/L The patient's anion gap is _________

25 or 25 mEq/L or twenty five or twenty-five or twenty five mEq/L Explanation The traditional anion gap is calculated by using the following equation: AG = ([Na+] + [K+]) - ([HCO3-] + [Cl-]). This patient's anion gap is therefore (140 + 5) - (110 + 10) = 25 mEq/L. A normal range of the anion gap is between 12 and 16 mEq/L (although labs may differ slightly on normal range depending on the means of measurement), so this patient has an elevated anion gap.

Assuming the same setup and conditions as in the preceding question, to maintain a steady state concentration of 1 g/L and a constant volume of 4 L, ____ liters of solution with ____ concentration need to be added to the bucket daily. 1; 1 g/L 2; 1 g/L 1; 2 g/L 2; 2 g/L

2; 1 g/L Explanation In this system, steady state occurs when input equals elimination, given that there is no production. Since 2 L of fluid with 1 g/L of substance are eliminated per day, matching that with the identical input (2 L at 1 g/L) will maintain the bucket at steady state volume and concentration. Although adding 1 L at 2 g/dL will maintain the amount of the substance in the bucket, the volume of fluid added is less than the volume eliminated in that case and steady state would not be maintained.

The patient in the video is breathing gas that contains 50% oxygen. Assuming that the total pressure of the inspired gas is atmospheric pressure, what is the partial pressure of oxygen in the inspired gas? 160 mm Hg 50 mm Hg 110 mm Hg 380 mm Hg

380 mm Hg Explantion: Recall that the partial pressure of a gas is calculated as the concentration (percentage) of that gas multiplied by the atmospheric pressure (Pgas = Patm (Fgas)). Atmospheric pressure is 760 mm Hg. Thus the PO2 or partial pressure of oxygen in the inspired gas is 0.5 X 760 = 380 mmHg.

Using your knowledge of homeostasis and blood pressure maintenance, which of the following would be an inappropriate therapy for hypotension (low blood pressure)? A drug that increases stroke volume. A drug that decreases the heart rate. A drug that increases systemic vascular resistance.

A drug that decreases the heart rate. Explanation Recall the "Ohm's Law analogy" equation for blood pressure (P): MAP - CVP = Q x SVR. Cardiac output (Q) is determined by both stroke volume and heart rate, specifically Q = SV x HR, meaning we could rewrite the equation as ΔP = SV x HR x SVR. To increase the difference in pressure, i.e. increase the mean arterial pressure, SV, HR, and/or SVR must increase. Any drug that decreases heart rate could worsen a patient's hypotension.

Which patient is LEAST likely to develop a buildup of metabolic acid? A healthy, poorly-oxygenated man using carbohydrates as his primary source of energy. A healthy, well-oxygenated man using carbohydrates as his primary source of energy. A healthy, well-oxygenated man using protein as his primary source of energy. A healthy, well-oxygenated man using fat as his primary source of energy.

A healthy, well-oxygenated man using carbohydrates as his primary source of energy. Explanation Aerobic metabolism of carbohydrates produces CO2, which forms carbonic acid (a respiratory or volatile acid). If the man is using other sources of fuel, e.g. protein or fat, or anaerobically metabolizing carbohydrates, the consequence is production of metabolic acids, such as SO4, PO4, lactate, or keto acids. Those acids must be excreted by the kidneys or metabolized in the liver and may build up if production exceeds elimination.

Inhalation is the process of bringing air into the lungs and exhalation is air leaving the lungs. Inhalation is considered an active process while exhalation, during resting breathing, is considered a passive process. With what you know about elastic recoil and surface tension, pick the statement(s) that are true. Inhalation is an active process because it takes muscle force to expand the lungs and overcome elastic recoil, moving the lungs from their resting position. Inhalation is an active pr

All of the above Explanation All of the choices are correct. Inhalation involves using respiratory muscles to overcome the elastic and surface forces of the lung. Upon relaxation at the end of a normal inhalation, both types of recoil forces contribute to returning the lungs to their resting position and moving air out of the lungs.

A 66-year-old female arrives in the emergency department complaining of headache, loss of appetite, and fatigue. Based on bloodwork, you find that she has an anion-gap metabolic acidosis. Which of the statements below best describes the processes in the kidney that will eliminate the metabolic acids that have built up in her blood? Anions are filtered out, protons are secreted, and bicarbonate is reabsorbed into the bloodstream. Cations are filtered out, protons are secreted, and bicarbonate is

Anions are filtered out, protons are secreted, and bicarbonate is reabsorbed into the bloodstream. Explanation An anion-gap metabolic acidosis is caused by the production of acids that dissociate into protons and anions that are not HCO3- or Cl-. These anions must be filtered into urine in order to be excreted from the body. The protons being produced must also be secreted from the cells lining the renal tubule, and for each proton secreted by the renal tubule cells, a molecule of HCO3- is absorbed into the bloodstream due to the mechanism of carbonic anhydrase.

A 72-year-old man is sitting comfortably in his chair receiving oxygen via nasal cannula (a device to deliver supplemental oxygen via the nose) at 2 L/min. He is on a normal diet and is in no distress. What do you expect his major metabolic waste products to be if his body is primarily metabolizing carbohydrates? lactic acid keto acids SO4, PO4 CO2

CO2 Explanation Aerobic metabolism of carbohydrates produces CO2. At rest, most individuals rely on a mix of fuel sources but produce approximately 200 ml/min of carbon dioxide from aerobic metabolism of carbohydrate. Protein metabolism produces PO4 and SO4 while fat metabolism produces keto acids and anaerobic glucose metabolism produces lactic acid. Note: there is a baseline level of metabolism of fats and proteins (especially in between meals).

Consider a patient with a condition called septic shock, due to a systemic infection. As a result of this condition, the resistance of the systemic circulation of the cardiovascular system has decreased significantly. What do you predict happens to the overall flow (cardiac output) if the driving pressure (the ΔP of the system) stays the same? Cardiac output stays the same. Cardiac output increases. Cardiac output decreases.

Cardiac output increases. Explanation: Ohm's Law tells us that flow is equal to driving pressure divided by resistance. If resistance is reduced by chemicals related to the infection, which cause the blood vessels to dilate (i.e., have a larger radius), and driving pressure (mean arterial pressure minus central venous pressure) stays the same, then you would expect flow (cardiac output) to increase.

(NOTE: This is not a duplicate question) Carbon monoxide, a product of combustion, is a toxic gas that has an extremely high affinity for hemoglobin (much higher than that of oxygen for hemoglobin); consequently, as soon as it dissolves in the liquid part of blood at low partial pressure, it diffuses quickly into red blood cells and binds to hemoglobin. In carbon monoxide (CO) poisoning, even with very low partial pressure of inspired CO, CO rapidly binds to hemoglobin (Hgb), leaving a lower fra

Decreased oxygen content Explanation: Recall that the O2 content of the blood is the sum of the dissolved oxygen as well as the oxygen bound to Hgb. The O2 content of the person's blood would be low, given that the amount of O2 bound to Hgb is decreased by CO binding. Given that the majority of blood oxygen content is normally comprised of oxygen bound to Hgb, this person would likely suffer from tissue hypoxemia (low oxygen in the tissues of the body).

Consider a house with temperature controlled by a thermostat connected to a furnace. Someone opens a window in the house, thereby impacting the temperature. In the human body, blood pressure is controlled by a homeostatic mechanism. In the body-blood pressure system, what might be the equivalent of opening a window in the house-temperature system? Surgically removing the baroreceptor such that it no longer sends a signal to increase contractility. The baroreceptor signalling the heart to increa

Cutting an artery, leading to a decrease in blood pressure. Explanation In the house example, someone opens a window, which leads to a drop in temperature in the house that is sensed by the thermostat, which then sends a signal to the furnace to increase heat production. The opening of the window is not directly sensed by the thermostat - rather, it is the stimulus that leads to the change in temperature. The answer choice that best represents a stimulus that leads to a change ultimately sensed by the baroreceptor is the cutting of the artery, which leads to bleeding, loss of blood from the vascular space, and a fall in blood pressure.

If systemic vascular resistance (SVR) goes up and mean arterial pressure (MAP) does not change, what happens to oxygen delivery (DO2)? DO2 stays the same DO2 decreases DO2 increases

DO2 decreases Explanation DO2 is determined by the oxygen content of blood and the volume of blood delivered per unit of time. Increased vascular resistance with no change in pressure means that cardiac output has decreased, therefore decreasing the volume of blood delivered to the body per unit time.

In a person with a low blood protein level, why might it be difficult to perform hemodialysis to remove fluid and electrolytes? Dialysis might remove the little protein that the patient has left. Dialysis might trigger a pro-inflammatory reaction based on reactivity associated with low blood protein states. Dialysis would draw fluid from the intravascular space and blood pressure might fall, given that the low blood protein will impair equilibration with the interstitial space.

Dialysis would draw fluid from the intravascular space and blood pressure might fall, given that the low blood protein will impair equilibration with the interstitial space. Explanation: Dialysis draws fluid from the intravascular space. Under conditions of normal protein levels, the intravascular space will equilibrate with the interstitial space during dialysis, with the result that fluid is also removed from the interstitial space via the intravascular space. With insufficient blood protein, equilibration between intravascular and interstitial space is impaired, and any fluid drawn from the intravascular space may drop blood pressure into a dangerous range, given the lack of intravascular oncotic pressure that would normally draw fluid from the interstitial space to the intravascular space.

How is Ohm's Law altered under conditions of turbulent flow relative to laminar flow? Consider what you know about the relationship between ∆P and flow. It is not altered. Flow is equal to resistance divided by driving pressure. Driving pressure is equal to 2 x Flow x Resistance. Driving pressure is proportional to Flow2.

Driving pressure is proportional to Flow2. Explanation: The Ohm's Law analogy for fluid flow states that the driving pressure, or ∆P, equals flow times resistance. In other words, ∆P is proportional to flow. You learned that under conditions of turbulent flow, ∆P is proportional to flow squared, and thus, the linear proportionality between ∆P and flow of Ohm's Law no longer applies.

As respiratory quotient increases, ventilation increases. What would happen if this increase in ventilation did not occur? (select all that apply) Enzyme function would be altered. Excess CO2 would accumulate. Blood pH would be lowered. Cardiac output would fall.

Enzyme function would be altered. Excess CO2 would accumulate. Blood pH would be lowered. Explanation An increase in the respiratory quotient implies a higher production of CO2. Some of this CO2 will dissolve in the blood and combine with water to form carbonic acid, and some of this carbonic acid will dissociate into bicarbonate and protons. Increased ventilation expels excess CO2, thereby preventing a drop in blood pH and maintaining the bicarbonate buffering system's capacity to maintain pH at a level safe for cellular enzymes.

If a patient has gained several liters of fluid between dialysis treatments, why can't we remove that fluid quickly (e.g. in a matter of minutes)? The dialysis machine cannot generate sufficient pressures to draw off fluid this quickly. Fluid is being removed from the intracellular compartment, and cells' osmolarity would undergo a potentially dangerous change. Fluid is being removed from the intravascular space, and time is needed for the intravascular space to equilibrate with the interstitia

Fluid is being removed from the intravascular space, and time is needed for the intravascular space to equilibrate with the interstitial space. Explanation: It takes time for the intravascular space to equilibrate with the interstitial space. Fluid is being drawn from the intravascular space and with time for equilibration, from the interstitial space as well. If performed quickly, dialysis may draw fluid primarily from the intravascular space and blood pressure may drop.

Thermostats are not perfect. The temperature must fall slightly below the setpoint to activate a system that turns on the furnace. The system often overshoots the setpoint after the furnace turns on. Which graph best represents the temperature readings for a thermostat set to 70 degrees F? Graph A - the temperature remains constant. Graph B - the temperature fluctuates above and below the setpoint. Graph C - the temperature rises above the setpoint, then drops back to the setpoint. Graph D - th

Graph B - the temperature fluctuates above and below the setpoint. Explanation A home heating system (a furnace controlled by a thermostat) is a simple negative feedback loop. Many control systems in the human body are also negative feedback loops. The body continually makes miniscule adjustments to stay as close to the setpoint as possible. Maintaining the values at exactly the setpoint is usually impossible, and therefore the system both overshoots and undershoots the setpoint. Therefore the graph of a negative feedback loop, such as a thermostat, would look like a sine wave.

In the following five acid-base reactions, a substance that acts as either an acid OR a base is bolded. Select reaction(s) where the indicated compound acts as an acid. HCO3- + H3O+ ⇌ H2CO3 + H2O HCO3- + H3O+ ⇌ H2CO3 + H2O H2O ⇌ H+ + OH- H2O + H+ ⇌ H3O+ X- + H+ ⇌ XH

HCO3- + H3O+ ⇌ H2CO3 + H2O H2O ⇌ H+ + OH- Explanation For our purposes, a substance is considered an acid when it donates protons (H+), and a base when it accepts protons. In the third reaction, H2O donates a proton, meaning it acts as an acid, while in the fourth reaction, it accepts a proton and acts as a base. The first and second options show the same reaction, with a different compound bolded: bicarbonate (HCO3-) acts as a proton acceptor (base) while carbonic acid (H2CO3) acts as a proton donor (acid). In the final reaction, "X-" acts as a proton acceptor, and therefore X is a base.

Claudia is a 20-year-old female who presents to your clinic complaining of fatigue. You order a blood panel, which reveals that her hemoglobin (Hgb) is 9 g/dL, which is much lower than normal. Based on this result, you conclude that Claudia has anemia (a deficiency of red blood cells or hemoglobin). How is her oxygen carrying capacity altered relative to her normal Hgb level of 12 g/dL? (Note: One gram of Hgb carries 1.39 mL of oxygen; assume that the contribution of oxygen dissolved in blood is

Her oxygen carrying capacity is decreased by 25%. Explanation The carrying capacity of the blood is determined primarily by the concentration of Hgb and the oxygen carrying capacity of Hgb. We are assuming that the carrying capacity of Hgb remains constant; because we are calculating a change relative to normal, this constant cancels out when calculating the relative oxygen carrying capacity of the blood. Thus: Relative carrying capacity = 9 g/dL / 12 g/dL = 0.75. 100% - 75% = 25%. In other words, the oxygen carrying capacity of her blood is reduced by 25%.

During exercise, the human body relies heavily on the lungs and the cardiovascular system to meet its increased energy demands while maintaining the buffering capacity of the blood. Your friend Horton is running as fast as he can "until he drops". He takes no medications. In which of the following scenarios is Horton MOST likely to be limited by his lungs in his ability to exert himself? Horton is an elite athlete who has developed mild cardiac dysfunction. Horton is out of shape and has mild e

Horton is fit and has severe exercise-induced asthma. Explanation A normal, healthy individual will not utilize their full lung capacity, even during intense exercise. Therefore, if Horton were of average fitness with no underlying pulmonary dysfunction, the lungs would not be expected to be his limiting factor. Elite athletes may approach full use of reserve lung capacity during exercise, but if Horton had any sort of cardiac dysfunction, this would more likely be his limitation. In the remaining two options, Horton has exercise-induced asthma, which will diminish his pulmonary capacity during an attack. In one case, Horton is in good shape, while in the other, he is out of shape. As a person increases physical fitness with training, the heart's ability to pump blood and deliver oxygen to the tissues increases, as well as the ability of the tissues to utilize that oxygen. Therefore, the constricted airways during an asthma attack are more likely to be the limiting factor in an indivi

Starling forces govern the movement of water and electrolytes across capillaries. How do hydrostatic and oncotic (osmotic) forces change as blood moves along the length of each capillary? Hydrostatic pressure and oncotic pressure both decrease along the capillary. Hydrostatic pressure increases and oncotic pressure decreases. Hydrostatic pressure and oncotic pressure both increase along the capillary. Hydrostatic pressure decreases and oncotic pressure stays roughly the same.

Hydrostatic pressure decreases and oncotic pressure stays roughly the same. Explanation: The vascular wall is minimally permeable to protein. Since the hydrostatic pressure is greater than in the arterial side of the capillary than in the interstitium, water will move out of the capillary. The protein left behind in the capillary becomes more concentrated as water exits to the interstitium, but the change in protein concentration is almost unmeasurably small, given that the amount of water leaving the intravascular space is small. Hydrostatic pressure decreases along the length of the capillary.

As air moves out of a flexible structure through a tube, as shown in the figure above, what do you expect to happen to the pressure along the length of the tube from the source to the opening? What would you expect if the tube was suddenly closed and flow stopped? Think about the principles behind rapid oscillation. In the open tube, the pressure would gradually decrease along the length of the tube, while in the closed tube, the pressure would be equal along the length of the tube. In the open

In the open tube, the pressure would gradually decrease along the length of the tube, while in the closed tube, the pressure would be equal along the length of the tube. Explanation When the tube has air flowing through it, i.e. under dynamic conditions, you expect the pressure to decrease along the length of the tube due to loss of energy and other factors. Importantly, ΔP Flow for laminar flow, and ΔP is defined as P1 - P2, where P1 is the pressure nearer the source of the flow and P2 is the pressure further from the source of flow (e.g. the pressure difference between an alveolus and the mouth or inside a balloon and the opening at the neck of the balloon). Factors that cause flow to decrease, e.g., increased resistance in the tube, lead to greater drops in pressure along the tube until reaching the tube's opening where the pressure is equal to the surrounding pressure. When the tube is closed and there is no flow, i.e. under static conditions, pressure will equalize within the t

If resistance increases along the length of a tube and outside pressure does not change, what happens to the transmural pressure along the length of the tube? Increased resistance in the tube causes increased inside pressure, causing the transmural pressure to rise. Increased resistance in the tube causes increased outside pressure, causing the transmural pressure to drop. Increased resistance in the tube causes decreased inside pressure, causing the transmural pressure to drop. Increased resis

Increased resistance in the tube causes decreased inside pressure, causing the transmural pressure to drop. Explanation Recall that ΔP = flow× resistance. As energy is lost in overcoming resistance, the pressure inside the tube decreases. If the outside pressure remains constant, the drop in internal pressure will cause a drop in transmural pressure (PTM = Pin - Pout), eventually leading to tube collapse once PTM < Pcrit.

How does surfactant act to reduce the inward collapsing forces on the alveoli? It reduces the elastic recoil forces. It disrupts Van der Waals forces. It provides an outward pressure that counteracts the collapsing forces. It adds a positive repelling charge.

It disrupts Van der Waals forces. Explanation Surfactant molecules have hydrophobic and hydrophilic regions. By interposing their hydrophilic regions in the air-water interface in alveoli, the surfactant molecules disrupt the surface forces.

Describe the acidosis that is occurring in the patient in the question above. What might have caused the increase in his anion gap? Respiratory acidosis characterized by fat and protein breakdown without adequate byproduct excretion. Respiratory acidosis characterized by aerobic carbohydrate breakdown without adequate byproduct excretion. Metabolic acidosis characterized by aerobic carbohydrate breakdown without adequate byproduct excretion. Metabolic acidosis characterized by protein breakdown

Metabolic acidosis characterized by protein breakdown without adequate byproduct excretion. Explanation We know that this patient has a metabolic acidosis because of the presence of an elevated anion gap of 25 mEq/L (normal range: 12-16 mEq/L). In addition, the low serum HCO3- confirms the presence of the metabolic acidosis, and is likely due to the buffering of the acid. Finally the low PCO2 tells us that this is not a respiratory acidosis. The patient's anion gap is explained by the presence of unmeasured serum anions. This patient is likely producing acid, and therefore anions, without adequate renal clearance due to his kidney dysfunction. Protein and fat metabolism both produce acids (protein metabolism leads to formation of PO4- and SO4-, while fat metabolism leads to ketones). The kidneys are the primary means of removing phosphoric and sulfuric acid, the byproducts of protein metabolism, from the blood.

A patient with chronic obstructive pulmonary disease (COPD) who is mildly short of breath at baseline goes on a carbohydrate-only diet. How would you expect her minute ventilation to change? Recall that minute ventilation = respiratory rate [breaths/min] x tidal volume [difference in volume of air between normal inhalation and exhalation]. Minute ventilation increases. Minute ventilation decreases. Minute ventilation stays the same.

Minute ventilation increases. Explanation Since the respiratory quotient (R = ratio of production of CO2 to consumption of O2) is highest for a high-carbohydrate diet (R =1) compared to a high-protein (R = 0.8) or high-fat (R = 0.7) diet, more carbon dioxide is produced by the metabolism of carbohydrate-only fuel (vs. protein/fats/mixed diet). Thus, if ventilation were not able to increase to remove the excess CO2 produced, PaCO2 would rise and pH would fall due to the dissociation of carbonic acid into a proton and a molecule of bicarbonate.

Antidiuretic hormone (ADH) is a molecule that partly acts on the nephron to increase water permeability at the collecting duct (the distal portion of the renal tubule). The concentration of solutes is higher in the fluid around this portion of the renal tubule than in baseline conditions. Predict what will happen to water reabsorption in the kidney and the concentration of solutes in the urine when ADH is present? More water will be reabsorbed and the urine solutes will be more concentrated. Le

More water will be reabsorbed and the urine solutes will be more concentrated. Explanation: Water moves from regions of low solute concentration to high solute concentration. As water exits the renal tubule, the urine becomes more concentrated.

The main airway (trachea) leads into a system of multiple smaller airways and alveoli. Compare the effect of a 50% airway narrowing in the trachea versus a 50% narrowing in one of the smaller airways. Narrowing at the level of the smaller airway will have a larger impact since it is in a parallel system. Narrowing at the level of the trachea will have a larger impact since it is in a series system. Narrowing at the level of the smaller airway will have a larger impact since it is in a series sy

Narrowing at the level of the trachea will have a larger impact since it is in a series system. Explanation: The total resistance of tubes arranged in series is the sum of the resistances of each tube. In contrast the total resistance of tubes arranged in parallel is the inverse of the sum of 1/R of each of the individual tubes. Consequently, the total resistance is less than the individual resistance of any given tube when they are arranged in parallel. The small airways are arranged in parallel; thus, a narrowing of a small airway will have less impact on resistance of the total system than a narrowing of the trachea.

Carbon monoxide, a product of combustion, is a toxic gas that has an extremely high affinity for hemoglobin (much higher than that of oxygen for hemoglobin); consequently, as soon as it dissolves in the liquid part of blood at low partial pressure, it diffuses quickly into red blood cells and binds to hemoglobin. In carbon monoxide (CO) poisoning, even with very low partial pressure of inspired CO, CO rapidly binds to hemoglobin (Hgb), leaving a lower fraction of oxygen binding sites on Hgb avai

Normal PO2 Explanation: The person's arterial PO2 is likely to be normal. Remember that the O2 content of the blood consists of dissolved O2 and O2 bound to Hgb. The partial pressure of O2 in the blood is determined by the O2 dissolved in the liquid portion of the blood and does not depend on the amount of Hgb or the availability of oxygen binding sites on Hgb. The partial pressure of inspired CO is usually low in CO poisoning and does not change the PO2 in the inspired air; therefore, oxygen will reach the same equilibrium between its gaseous state in the alveolus and its dissolved state in the alveolar capillary blood during carbon monoxide poisoning. However, oxygen content (dissolved oxygen + oxygen bound to hemoglobin) decreases because of carbon monoxide's interaction with hemoglobin (taking up oxygen binding spots). This interaction also causes hemoglobin-bound oxygen to be more tightly bound, thereby impairing oxygen release at the tissues. These factors contribute to decrease

An individual develops anemia (low red blood cell counts). During moderate exercise, lactic acid accumulates in her muscles. This is primarily a consequence of a reduction in arterial partial pressure of O2 (PaO2). mean arterial pressure (MAP). systemic vascular resistance. O2 content of the blood. cardiac output.

O2 content of the blood. Explanation A reduction in red blood cell counts means that the blood can carry less oxygen. Total hemoglobin is reduced. Lactic acid arises from anaerobic metabolism, suggesting insufficient supply of oxygen to meet normal/physiological range demand during exercise. The insufficient supply is likely due to low oxygen content in the blood due to anemia. Her arterial partial pressure of O2 (PaO2) should be normal, even with anemia (review the partial pressure lesson if this is unclear).

What forces contribute to the water balance between the intravascular space and the interstitial space? Oncotic and hydrostatic Osmotic only Oncotic only Hydrostatic only

Oncotic and hydrostatic Explanation: Movement of fluid between the vascular and interstitial compartments is governed by Starling forces - hydrostatic and oncotic forces.

In special kidney capillaries called glomerular capillaries, hydrostatic pressure stays high along the length of the capillary such that a great deal of fluid is filtered (pushed out of the vessel) along the length of the capillary (much more than in systemic capillaries). What do you expect happens to the oncotic pressure along the length of the glomerular capillary? Oncotic pressure falls because protein also moves out of vessels. Oncotic pressure rises because fluid moves out of vessels. Onc

Oncotic pressure rises because fluid moves out of vessels. Explanation: In the systemic capillary, oncotic pressure stays almost the same along the capillary. In the kidney, however, there are special mechanisms to minimize the fall in hydrostatic pressure as water is filtered from the capillary. Thus, the oncotic pressure rises along the entire length of the capillary.

The Fick principle, as applied to the heart, describes the relationship between cardiac output, oxygen consumption, and oxygen concentration in the blood. The equation below is a mathematical representation of this relationship. Select the appropriate symbol for each of the labeled terms in this equation. ____(Cardiac Output) (______ (Arterial O2 content) - _____ (Venous O2 content) ) = ______

Q(CaO2-Cv02)=V02

Now that you have tied off the two balloons in the previous question, you wonder how to make the transmural pressure equal for both balloons. You remember that putting a balloon under water decreases the volume of the balloon. Which diagram below depicts the relative depths of each balloon to achieve a similar transmural pressure across both balloons? (Note: depths of balloons are not meant to indicate absolute values, just relative values). Assume you can connect the balloons to a machine that

Option A - The stiff balloon is deeper than the flexible balloon. Explanation Because you are not allowing the pressure inside the balloons to change, you must increase the pressure outside the stiff balloon to make its PTM smaller, i.e., bring it down to the PTM of the flexible balloon. By immersing the balloons underwater, you increase Pout in proportion to depth below the surface. This will tend to decrease the transmural pressure across both balloons and result in a decrease in the volume of the balloons. Recall that the stiff balloon has a higher positive transmural pressure at atmospheric pressure than the floppy balloon (owing to the higher Pin) when they are the same volume. Thus, we must immerse the stiff balloon at a greater depth (greater Pout) than the floppy balloon to achieve equal transmural pressures.

The respiratory quotient (R) is a term that is also found in an equation called the Alveolar Gas Equation, which calculates the pressure of oxygen in the alveolus (PAO2) taking into account the equilibrium between the amount of inspired oxygen and amount of oxygen entering the surrounding capillaries; if an individual is breathing room air, the equation can be approximated as: PAO2 = 150 - PCO2/R During intense exercise, assuming arterial pressure of CO2 is unchanged, what would you expect to ha

PAO2 increases during exercise. Explanation R increases as exercise increases in intensity, due to increasing utilization of carbohydrates for fuel. According to the alveolar gas equation, PAO2 increases when R increases. More conceptually, the alveolar oxygen concentration depends on the balance between transfer of oxygen from the alveolus to the blood and replenishment of alveolar oxygen via ventilation. With increasing intensity of exercise, replenishment of alveolar oxygen exceeds removal of oxygen via the blood and alveolar oxygen levels rise.

Imagine you are on the planet Venus where the atmospheric pressure is ~92 atm (1 atm = 760 mmHg). What are the partial pressures of the component atmospheric gases, given gas fractions of 96.5% CO2 and 3.5% N2? PCO2 96.5 atm; PN2 3.5 atm PCO2 88.8 atm; PN2 3.2 atm PCO2 0 atm; PN2 78 atm PCO2 73.3 atm; PN2 26.6 atm

PCO2 88.8 atm; PN2 3.2 atm Explanation: Partial pressure of a gas is calculated as the concentration (percentage) of that gas multiplied by the atmospheric pressure (Pgas = Patm (Fgas)). For CO2 on Venus, the calculation would be PCO2 = 0.965 X 92 = 88.8 atm; for N2, PN2 =.035 X 92 = 3.2 atm. The partial pressures of the component gases added together equals the total pressure in the system, in this case 92 atm.

As climbers ascend Mount Everest, the atmospheric pressure drops from 1 atm to ~0.333 atm (1 atm = 760 mm Hg) at the summit. Calculate the partial pressures of oxygen and nitrogen at the peak of Mount Everest. PN2 0.78 atm; PO2 0.21 atm PN2 0.26 atm; PO2 0.07 atm PN2 0.07 atm; PO2 0.26 atm PN2 0.21 atm; PO2 0.78 atm

PN2 0.26 atm; PO2 0.07 atm Explanation: This is calculated by the percentage of a gas multiplied by the atmospheric pressure. Given that only total pressure changes as one ascends, but not the fractions of O2 (21%) and N2 (78%), we calculate 0.333 X 0.78 = 0.26 atm of nitrogen; 0.333 X 0.21 = 0.07 atm of oxygen.

In an airway with some supportive architecture, at what level of transmural pressure does airway collapse occur? PTM > Pcrit PTM < Pcrit PTM > 0 PTM < 0

PTM < Pcrit Explanation In an airway with supportive architecture, there will be tube narrowing as soon as the transmural pressure reaches zero and begins to become negative. However, the structure of the airway will prevent its complete closure at this point. Further down the airway, at a certain negative transmural pressure that we call Pcrit or critical pressure, the airway will close because the negative transmural pressure is sufficient to overcome the stabilizing force of the airway's supportive architecture. Note that in this case, PTM is more negative (less than) Pcrit.

You have two balloons that are the same size when uninflated. One is made out of thick, stiff rubber (the "stiff balloon") and one is made of thin, flexible rubber (the "floppy balloon"). You inflate the balloons with helium until they reach some equal volume. How will the transmural pressure of the balloons compare? PTM floppy balloon = PTM stiff balloon PTM floppy balloon > PTM stiff balloon PTM floppy balloon < PTM stiff balloon

PTM floppy balloon < PTM stiff balloon Explanation Transmural pressure (PTM = Pin - Pout) dictates the final volume of a flexible structure. However, note that both balloons are not equally elastic. At any volume, the stiff balloon will have higher elastic recoil forces than the floppy balloon. This means that the stiff balloon requires a higher transmural pressure to maintain the same volume as a more flexible balloon.

Certain athletic training routines involve repeated bursts of high intensity activity followed by a brief recovery period. This is accompanied by a burning sensation in the working muscles and a drop of pH in the muscle tissue. What metabolic events best explain this situation? Production of lactic acid due to anaerobic metabolism. Production of lactic acid due to aerobic metabolism. Production of CO2 due to anaerobic metabolism. Production of CO2 due to aerobic metabolism.

Production of lactic acid due to anaerobic metabolism. Explanation High intensity muscular activity over a short period of time requires energy from anaerobic metabolism. The primary byproduct of anaerobic metabolism in humans is lactic acid.

During prolonged exercise, carbohydrate stores are eventually depleted, and the body begins to increasingly utilize proteins and lipids as a fuel source. What might you expect to happen to the respiratory quotient during this shift? R would increase. R would decrease. R would remain constant.

R would decrease. Explanation Respiratory quotient (R) is dependent on the fuel source used, with pure carbohydrate use leading to an R of 1.0; proteins, 0.8; lipids, 0.7. As the fuel source switches from carbohydrates to lipids, the respiratory quotient therefore decreases.

In the same animal experiment above, you now measure the pH of the blood 6 days after injection. Recall that Rat 1 has normally functioning kidneys and normally functioning lungs. Rat 2 has normally functioning kidneys and poorly functioning lungs. Rat 3 has poorly functioning kidneys and normally functioning lungs. Rank the rats according to the change in pH from baseline pH in each individual rat: Rat 1 Δ pH = Rat 2 Δ pH = Rat 3 Δ pH Rat 1 Δ pH < Rat 2 Δ pH < Rat 3 Δ pH Rat 1 Δ pH < Ra

Rat 1 Δ pH = Rat 2 Δ pH < Rat 3 Δ pH Explanation 6 days after injection, the functioning capacity of the kidneys will be the primary determinant of Δ pH. The rats with functioning kidneys (1 & 2) will have a small, nearly equivalent Δ pH; the rat with poorly functioning kidneys (3) is unable to eliminate acid and will have a larger Δ pH. Compensation of one organ for the other reduces the severity of the pH abnormality but doesn't correct pH fully to 7.40.

In an animal experiment, three rats are intravenously given a small amount of a metabolic acid that is excreted via the kidneys. Rat 1 has normally functioning kidneys and normally functioning lungs. Rat 2 has normally functioning kidneys and poorly functioning lungs. Rat 3 has poorly functioning kidneys and normally functioning lungs. You check the pH of the blood in the rats 30 minutes after injection. Rank the rats according to the magnitude of the change in pH from baseline pH in each indivi

Rat 1 Δ pH = Rat 3 Δ pH < Rat 2 Δ pH Explanation An injected metabolic acid can be immediately buffered by HCO3- in the bloodstream, which leads to the production of CO2 that is eliminated on the order of seconds to minutes by the lung. The acid is also eliminated on the order of days by the kidney. Thus, a mere 30 minutes after injection, the functioning capacity of the lungs will be the primary determinant of Δ pH. The rats with functioning lungs (1 & 3) will have a small, nearly equivalent Δ pH; the rat with poorly functioning lungs (2) is unable to eliminate acid quickly and will have a large Δ pH.

When thinking about the total input for the human body, it is important to consider not only what a person eats or drinks, but also what is produced by cellular metabolism. Creatinine, a waste product produced by muscle cells, is excreted by the kidneys, and measuring serum creatinine allows us to assess renal function. Normal serum creatinine is less than 1.0 mg/dL, but can rise acutely if kidney function decreases. A 56-year-old woman comes to the emergency department with a serum creatinine c

Renal function has declined, resulting in decreased creatinine excretion. Explanation Since the serum concentration of creatinine has increased, there must have been an increase in input or decrease in output. In this woman, it is unlikely that there is input of creatinine (from an external source). Therefore, there must be decreased creatinine excretion. Given that the kidneys are the primary route of excretion for creatinine, renal function must have declined if less creatinine is being excreted.

If the lungs do not function to eliminate sufficient CO2, which of the following might you expect to occur? Hemoglobin proteins in the red blood cells will become less protonated. Serum bicarbonate levels will increase. Respiratory alkalosis will occur. Metabolic acidosis will occur.

Serum bicarbonate levels will increase. Explanation CO2 in the bloodstream diffuses into red blood cells, where an enzyme catalyzes its reaction with water to form carbonic acid, which dissociates into bicarbonate and a proton. CO2 + H2O ⇌ H2CO3 ⇌ HCO3- + H+ Negatively charged proteins contained in the red blood cells, like hemoglobin, can bind protons, buffering the respiratory acid, and leaving the remaining bicarbonate to diffuse out of the cell and increase the bicarbonate concentration in the blood. A dysfunctional respiratory system that does not eliminate enough CO2 will therefore lead to more protonated hemoglobin proteins, an increase in serum bicarbonate levels, and results in respiratory acidosis (as opposed to metabolic acidosis).

Which of the following is a reasonable explanation for why patients with emphysema are more likely to experience airway collapse than patients with healthy lungs? Emphysema patients are unable to generate a sufficient thoracic pressure during inhalation to keep airways open. Small airways have lower resistance, causing an increase in pressure inside airways, resulting in airways collapse. Small airways have weaker elastic support, making them more prone to collapse when the transmural pressure

Small airways have weaker elastic support, making them more prone to collapse when the transmural pressure becomes negative. Explanation Airway collapse will occur when the transmural pressure of the airways becomes negative, i.e. when the external pressure exceeds the internal pressure. In order for that to occur, the structure must also be infinitely flexible. In most airways, however, there are fine ligamentous structures that hold airways open, but these structures are often destroyed in patients with emphysema. These floppier airways are consequently more prone to collapse when the transmural pressure becomes increasingly negative. A higher thoracic pressure may increase flow, but it would also increase the external pressure on the airway, leading to a decreased transmural pressure. Sturdier airways would make them less likely to collapse, as a more negative transmural pressure would be required to overcome the strength of the airway.

A balloon with a small radius would seem to have less collapsing force vs. a balloon with a larger radius (more inflated). How is this consistent with Laplace's Law, given that you learned in the video that bubbles at smaller radii have greater pressure than ones at larger radii? Tension and radius change at the same rate for a balloon, so that pressure inside the balloon remains constant over a range of radii. Tension for the different size balloons is not constant. The pressure inside the sma

Tension for the different size balloons is not constant. Explanation For the continuous soap bubble film, we assumed that the surface tension was the same for the bubbles of differing diameters. For the balloon, however, the wall tension increases as the balloon is inflated. The balloon's wall tension is governed by Laplace's Law, but the operative forces are due to the elastic recoil forces, rather than surface tension. For the balloon, wall tension increases as pressure and radius both increase.

Imagine you take a 750mL bottle of champagne (PCO2 ~ 5.5atm in the bottle) on a trip to Venus and, with considerable effort, uncork the bottle in celebration. The PCO2 in the atmosphere is 88.8 atm. Predict what will happen when the inside of the bottle is exposed to the Venusian atmosphere (assume no change in temperature). The PCO2 in the bottle will fall to zero as all the CO2 bubbles furiously out of the bottle. The PCO2 in the bottle will stay at 5.5 atm following equilibration and only mi

The PCO2 in the bottle will rise to 88.8 atm following equilibration Explanation: The PCO2 in the bottle at sea level on Earth was 5.5atm. Once the bottle was on Venus the PCO2 of the atmosphere is 88atm. The CO2 thus rapidly moves into the bottle based on the PCO2 gradient between the Venusian atmosphere and the bottle.

According to Laplace's Law, in a structure with an air-liquid interface, the smaller the radius, the greater the pressure (assuming surface tension is constant). Contrast that to a balloon. When you blow up a balloon, it's easier to blow up the balloon at low volumes vs. high volumes because: The balloon's elastic recoil is reduced at high volumes. The balloon is thicker at lower volumes. The balloon's compliance increases as the balloon's volume goes up. The balloon's compliance decreases as t

The balloon's compliance decreases as the balloon's volume goes up. Explanation As the balloon stretches, it requires a greater change in pressure to achieve a given change in volume. In other words, as the balloon is inflated, its compliance decreases (remember that delta P is in the denominator of compliance). Therefore, the balloon becomes harder to blow up as you inflate it further.

Consider a simplified model of the respiratory system, where each lung consists of a single air sac, called an alveolus (plural: alveoli). Carbon dioxide produced in the muscle tissue travels in the venous blood and is offloaded in the alveolus, and is subsequently breathed off. The blood then flows from the alveolar capillary into the arterial system with a lower carbon dioxide content than was in the venous blood. When one of the two alveoli in this model is blocked off (i.e., it is not being

The blood offloads more CO2 at the functional alveolus. Explanation When an alveolus is blocked off, the total CO2 elimination of the system initially goes down. Assuming production (and intake) remain constant, the system enters dynamic conditions as the systemic CO2 levels rise. As the CO2 levels in the blood rise, the concentration gradient between the blood and the functional alveolus increases. This facilitates increased offloading of CO2 into this functional alveolus, meaning that the gas exiting the alveolus has a higher PCO2 (the steep concentration gradient is reestablished with each breath). Once the elimination of CO2 in the functional alveolus has risen to match the production of CO2, the system enters a new steady state, at a higher level of CO2 than prior to the blockage.

The serum anion gap is a calculated laboratory value used to evaluate the etiology of acid-base disorders. Recall that the anion gap is represented with the following equation: Anion gap = ([Na+] + [K+]) - ([Cl-] + [HCO3-]) Note that many people do not include the concentration of K+ in their calculation because the body controls potassium concentration very tightly and it rarely varies outside the range of 3.5-4.5 mEq/L; if one uses K+ in the calculation, the "normal" value is up to 16; witho

The body has a net neutral charge, but the anions we measure have been replaced by anions other than Cl- and HCO3-. Explanation As stated in the video, the body is electrically neutral. The anion gap is a measured quantity and reflects the relative concentrations of the anions (Cl- and HCO3-) and cations (Na+ and K+) measured in routine blood tests, rather than the overall electrical charge of the body. It is calculated by subtracting the sum of the serum chloride and bicarbonate levels from the sum of serum sodium and potassium levels: Anion gap = ([Na+] + [K+]) - ([Cl-] + [HCO3-]). An increased anion gap can therefore theoretically result from an increase in the measured cation levels or a decrease in the measured anion levels. The only answer selection that represents one of these two possibilities is that measured anions are replaced with anions that are not measured and included in the calculation. If measured cations were replaced by unmeasured cations, there would be fewer tot

Why might a cell shift toward anaerobic metabolism? (select two answers) The cell cannot efficiently utilize delivered oxygen. The cell is using protein as an energy source. The cell is not receiving enough oxygen. The cell is using fat as an energy source. Anaerobic respiration is more efficient.

The cell cannot efficiently utilize delivered oxygen. The cell is not receiving enough oxygen. Explanation Cells must have both adequate oxygen delivery and adequate oxygen utilization to perform aerobic respiration. If either of these is disrupted or the cell's energy requirements exceed oxygen delivery or utilization, the cell may resort to anaerobic metabolism to supplement energy resulting from aerobic metabolism. Anaerobic respiration is not more efficient than aerobic respiration (aerobic respiration produces a net of approximately 38 ATP molecules for one glucose molecule, compared to only 2 ATP net per glucose consumed anaerobically), and neither fat nor protein metabolism would require a cell to shift to anaerobic respiration.

In infant respiratory distress syndrome (RDS), the patient's lungs have less surfactant and as such that they need very high pressures in order to inflate their lungs. Compared to infants with normal lungs: The compliance is higher due to increased surface tension. The compliance is higher due to increased elastic forces. The compliance is lower due to increased elastic forces. The compliance is lower due to increased surface tension.

The compliance is lower due to increased surface tension. Surfactant acts as a detergent and reduces surface tension forces in the lungs, allowing surface tension to decrease with the radius of alveoli. Without surfactant (or with low levels), surface tension will be higher than normal, resulting in less compliant lungs. Compliance is decreased because a given change in volume requires a greater change in pressure than for lungs with normal amounts of surfactant.

Suppose the gas packed in a champagne bottle is manipulated such that the total pressure of the gas is 760 mm Hg, with a PCO2 of 720 mm Hg. PCO2 in the atmosphere is approximately 0. When the cork is pulled, what do you expect to happen? The cork would fall limply on to the table without a "pop" sound The cork would fly off with high speed, making a large "pop" due to the high PCO2 pressure gradient Air rushes into the bottle, displacing the champagne and pushing it out of the bottle forcefully

The cork would fall limply on to the table without a "pop" sound Explanation: It is the total pressure difference that determines whether gas rushes in or out of the bottle. In this case, the total pressure of the gas inside the bottle is the same as the atmospheric pressure, so when the cork is pulled, it should be an anticlimactic soundless event.

Why do the lungs not decrease in volume even as you move the scuba diver deeper under the water? The diver is breathing pressurized gas, the pressure of which matches the surrounding water pressure. The scuba diver is holding his breath, so no air can get in or out and the volume stays the same. The diver is able to keep the lungs at a normal volume just using the muscles of the chest wall, without any assistance.

The diver is breathing pressurized gas, the pressure of which matches the surrounding water pressure. Explanation: The diver is breathing pressurized gas, the pressure of which matches the surrounding water pressure. As mentioned in the Dive Descent - Explained Video, without breathing pressurized gas matched by the scuba regulator to the surrounding water pressure, divers would be unable to exert sufficient force with muscles to overcome the surrounding water pressure. For example, trying to breathe through a long tube that extends to the surface while lying on the bottom of a pool would be futile; even at relatively shallow depths, the pressure exerted by the water on the chest wall would be too much.

A highly trained Olympic athlete is now performing the same exercise physiology test and does markedly better. Why is it that she is not as limited by her cardiovascular system during exercise? (select two answers) The highly trained athlete relies solely on anaerobic metabolism and therefore does not require as much oxygen. The highly trained athlete has less efficient oxygen utilization at the level of the tissues. The highly trained athlete has mitochondria with improved oxygen utilization.

The highly trained athlete has mitochondria with improved oxygen utilization. The highly trained athlete is able to deliver more oxygen to her tissues. Explanation The highly trained athlete will have both increased oxygen delivery and more efficient utilization of oxygen in the tissues. Training improves her cardiovascular capacity, enabling increased oxygen delivery. Training also improves her tissue's ability to utilize oxygen. This means that the highly trained athlete is less limited by her cardiovascular system than the average person. The ATP produced by highly trained athletes is no more potent than the ATP produced by anyone else. Highly trained athletes do not rely solely on anaerobic metabolism, as energy production would be insufficient for sustained exercise.

Every day we produce large amounts of CO2. This CO2 can combine with water, forming carbonic acid, and dissociate into H+ and HCO3- in our blood. Why don't we become acidemic (blood pH below normal range) on a regular basis? The liver metabolizes CO2 into useful substances that are later used by the body. The kidneys remove most of the produced H+ by excreting it into urine. The lungs have the capacity to breathe off large amounts of CO2. A weak acid is formed by the dissociation of carbonic ac

The lungs have the capacity to breathe off large amounts of CO2. Explanation Our lungs are very effective at eliminating CO2, preventing buildup of excess CO2 that would lead to generation of H+. We breathe off the equivalent of approximately 15,000 mmol of acid per day from our lungs. The kidneys, on the other hand, can only remove about 60-70 mEq per day. Carbonic acid is a weak acid (i.e. does not completely dissociate into HCO3- and H+), but still contributes to proton accumulation in the bloodstream. CO2 is not metabolized into anything useful in the body and is excreted instead. Note that different units are used (mmol vs. mEq) to allow direct comparison between the lungs and the kidneys in terms of amount of protons removed. Milliequivalents (mEq) are used in the case of the kidney because some acids give off multiple protons, and the idea of equivalents (or milliequivalents) takes charge into account. The important point is that the rate of respiratory acid generation and elim

A balloon's internal pressure has increased, but the balloon has not changed in size. What must have happened to the external pressure for this to be true? Assume that elastic recoil is negligible. The outside pressure has increased the same amount as the inside pressure. The outside pressure has increased more than the inside pressure. The outside pressure has remained the same. The outside pressure has decreased.

The outside pressure has increased the same amount as the inside pressure. Explanation Transmural pressure dictates the final volume of a flexible structure. For an object to undergo no change in volume, the transmural pressure must remain the same. If the internal pressure of a balloon has increased, but the balloon has not changed in size, there must have been a simultaneous increase in the external pressure. Since PTM = Pin - Pout, for PTM to remain constant, Pout must have increased (or decreased) the same amount as Pin.

When oxygen enters the alveolus, its partial pressure is ~160 mmHg. Capillaries come in contact with the alveolus, and gases are allowed to equilibrate. Prior to equilibration, what do you predict the PO2 in the capillary blood is relative to the PO2 in the alveolus? The partial pressure of oxygen in the capillary is the same as the alveolus because the blood is well oxygenated at all times The partial pressure of oxygen in the capillary is greater than that of the alveolus because the tissues

The partial pressure of oxygen in the capillary is less than that of the alveolus, allowing oxygen to move from alveolus to capillary Explanation: The tissues of the body utilize oxygen; hence the oxygen level (and thus the PO2) in the capillary blood flowing by the alveolus is lower than that of the alveolus, prior to equilibration. Oxygen moves down its pressure gradient, from higher PO2 (alveolus) to lower PO2 (capillary).

You are monitoring a patient undergoing an exercise physiology test. You notice that the minute ventilation begins to increase out of proportion to the oxygen consumption when she begins a particularly challenging part of the test. What is happening? The test must not be very difficult for this patient, as the rise in minute ventilation should always rise in proportion to the rise in oxygen consumption unless the activity is not very strenuous. The patient appears to have a lung dysfunction tha

The patient has reached her anaerobic threshold and is increasing her minute ventilation to compensate for the production of metabolic acids. Explanation When the minute ventilation begins to increase out of proportion to the oxygen consumption, it is a sign that the patient has reached her anaerobic threshold. Instead of increasing minute ventilation primarily to deliver oxygen to the alveolus, the patient is also increasing her minute ventilation to breathe off excess CO2. When she reaches her anaerobic threshold, her tissues begin to produce lactic acid via anaerobic metabolism. Protons from the acid combine with bicarbonate in the blood to form water and CO2. In order to compensate, the patient will breathe with a higher minute ventilation to expel the CO2 to maintain a blood pH that is closer to normal.

A patient arrives in the emergency department with a low blood pressure of 78/56. What is ONE plausible explanation? The pressure inside the heart is normal, while the pressure outside the heart is lower than normal. The pressure inside the heart is normal, while the pressure outside the heart is higher than normal. The pressure inside the aorta is higher than the pressure outside of the aorta. The pressure inside the heart is equal to the pressure outside of the heart.

The pressure inside the heart is normal, while the pressure outside the heart is higher than normal. Explanation As you learned, the greater the filling volume of the ventricle, the more cardiac output it can generate. If we want a nicely filled heart, then we want the heart to have a high transmural pressure. If the heart's transmural pressure is instead low, it will be less full and therefore will be less able to generate an adequate cardiac output. Since BP = Q x SVR, a low cardiac output would explain a low blood pressure. If the pressure inside the heart is normal, while the pressure outside the heart is higher than normal, the transmural pressure of the heart would be low (and the heart will be small and unable to generate an adequate cardiac output).

Imagine you have a tank of water with an opening at the bottom from which water exits the tank. Flow from a faucet provides a steady input of water to replace the water lost through the opening, such that the system is in steady state. Which of the following might result in the attainment of a new steady state if the rate of water influx from the faucet decreases? The pressure inside the opening at the bottom of the tank remains the same. The pressure inside the opening at the bottom of the tan

The pressure inside the opening at the bottom of the tank decreases. Explanation The rate of outflow now exceeds the sum of the inflow plus production, so the height of the column drops. In our simple model, the pressure inside the orifice at the bottom of the tank decreases as the height of the column of water decreases. Since flow is related to the difference in pressure between the inside of the orifice and the outside of the orifice, it is feasible that as the height of the water decreases, the difference in pressure, and thus the flow, decreases until a new steady state is reached.

An asthmatic is having an acute asthma attack, and the airway radius has gone from 10 mm to 5 mm. What has happened to the pressure needed to maintain the same flow through the airway as was attained prior to the narrowing? Assume laminar flow. The pressure needs to be 16 times previous. The pressure can be 16 times less than previous. The pressure needs to be 2 times previous. The pressure can be 2 times less than previous.

The pressure needs to be 16 times previous. Explanation: The resistance of a tube is determined largely by Poiseuille's Law, which tells us that resistance is inversely proportional to r4. A reduction in radius by a factor of 2 translates to an increase in resistance by 24 = 16 fold. Using the equation ∆P = flow x resistance, you can see that if resistance increases by 16 times and flow stays the same, ∆P must also increase by 16 times.

An asthmatic is having an acute asthma attack, with difficulty breathing due to narrowing of the airways. If the airway radius has gone from 9mm to 3mm, what is the corresponding change in resistance? The resistance in the airway has decreased by a factor of 81. The resistance in the airway has increased by a factor of 81. The resistance in the airway has increased by a factor of 3. The resistance in the airway has decreased by a factor of 3.

The resistance in the airway has increased by a factor of 81. Explanation: The resistance of a tube is determined largely by Poiseuille's Law, which tells us that resistance is inversely proportional to r4. R ∝ 1/r4, thus R1 ∝ 1/94 and R2 ∝ 1/34 or R1 ∝ 1/6561 and R2 ∝ 1/81. Therefore R2/R1 = r14/r24 = 94/34 = 34 = 81.

Joe and John are identical twins. John had a bleeding ulcer last week and is anemic with a hemoglobin level of 10 grams/100 ml of blood; Joe has a hemoglobin level of 14 grams/100 ml of blood (normal range is 13.5-17 grams/100 ml). Both are completely healthy, with the exception of John's recent ulcer. The partial pressure of oxygen in the twin's blood is: The same Higher in Joe than John Higher in John than Joe You need more information to make the determination.

The same Hemoglobin is a protein carried in the blood that is responsible for the large oxygen carrying capacity of the blood. As oxygen enters the blood some is carried on hemoglobin and some is dissolved in "solution" or the plasma component of blood. Partial pressure only measures dissolved oxygen in the bloodstream, not oxygen bound to hemoglobin. When we measure hemoglobin in blood it is measured as grams of the protein per 100 milliliters of blood volume. This question drives at the difference between oxygen content (i.e. dissolved oxygen + oxygen bound to hemoglobin) and partial pressure of oxygen (i.e. reflective of dissolved oxygen only). Partial pressure of oxygen in the blood is independent of the hemoglobin or oxygen carrying capacity of the blood. Thus regardless of the amount of hemoglobin in the blood the partial pressure of oxygen is the same between the twins.

It's New Year's Eve and you are eager to drink some champagne. Unable to wait until midnight, you open a bottle and pour yourself a glass. Your date comes by and tells you to recork the bottle, which you promptly do. Several minutes later the champagne is no longer "bubbling." The partial pressure of carbon dioxide in the bottle above the champagne is now: The same as in the partial pressure of CO2 in the atmosphere The same as the partial pressure of CO2 in the champagne Less than the partial

The same as the partial pressure of CO2 in the champagne Explanation: Carbon dioxide (CO2) is dissolved in solution under high pressures as in a champagne bottle. When the cork is removed the high pressure in the bottle begins to equalize with atmospheric pressure outside the bottle. As the pressure rapidly decreases in the bottle, CO2 begins to come out of solution and is represented as bubbles. When the cork is reinserted into the bottle, CO2 coming out of solution begins to accumulate as a gas in the bottle and the PCO2 in the gaseous state increases. When the bubbling ceases, the partial pressure of CO2 in the gaseous state in the bottle equals the PCO2 dissolved in the liquid (champagne). Consequently, the PCO2 in the bottle is greater than the PCO2 in the atmosphere (where it is essentially zero). We cannot know with certainty how much greater the PCO2 is in the gaseous phase in the bottle compared to the atmosphere.

Thinking back to the the pastrami sandwich scenario, what do you think you would find if you measured the sodium concentration in Max's blood by the end of his long day? The serum sodium concentration is above normal. The serum sodium concentration is below normal. The serum sodium concentration is normal.

The serum sodium concentration is normal. Explanation: Although the amount of sodium and water in Max's body may be increased by the end of the day based on his ingestion of salt and water, the serum sodium concentration is likely normal. Salt intake and the consequent slight rise in serum osmolarity stimulates thirst, resulting in water intake and ultimately maintaining normal serum sodium.

Premature infants may lack surfactant and are at risk of developing infant respiratory distress syndrome (RDS). What do you imagine will happen to the surface tension compared to infants with normal surfactant levels? The surface tension will be less than normal. The surface tension will be higher than normal. The surface tension will be unchanged.

The surface tension will be higher than normal. Explanation Surfactant acts as a detergent and reduces surface tension forces in the lungs, allowing surface tension to decrease with the radius of alveoli. Without surfactant (or with low levels), surface tension will be higher than normal.

Imagine that cardiac output drops below a normal level due to a chronic illness such as heart failure. How does the body compensate to avoid entering an anaerobic state? Tissues will extract less oxygen from the blood, therefore venous blood returns to the heart with a higher oxygen content. The tissues will extract more oxygen per liter of blood delivered, holding oxygen consumption constant. Systemic vascular resistance will decrease, reducing the pressure required to deliver blood to tissues

The tissues will extract more oxygen per liter of blood delivered, holding oxygen consumption constant. Explanation As a whole, the body's tissues compensate for reduced cardiac output by extracting more oxygen per volume of blood delivered. With low cardiac output, less volume of blood is delivered per minute, but oxygen consumption can be maintained by extracting more oxygen from each liter of blood delivered. This can compensate for low cardiac output to a point, but if cardiac output drops too low this compensation will not be sufficient. The heart muscle itself is an exception to the method of compensation via increased extraction; the left ventricle of the heart has high oxygen extraction at baseline and therefore primarily relies on other mechanisms to maintain oxygen consumption.

Why do the lungs decrease in volume as you move the diver deeper under the water? Carbon dioxide moves from the lungs into the blood. Air escapes from the lungs into the pleural space. The elastic recoil forces of the lungs increase. The transmural pressure decreases.

The transmural pressure decreases. Explanation As you move the diver deeper, for every 10m (~33 feet) under the water surface, approximately 1 additional atmosphere (atm) of pressure (1 atm = 760 mm Hg) is exerted by the surrounding water (shown as Pout = Water Pressure). As Pout increases, the lungs are compressed and Pin increases as well, although not as much as Pout. At any given depth, the lungs' volume will come into equilibrium at a volume for which the forces due to Pin balance the sum of the forces due to Pout and Pel (the pressure due to elastic recoil forces). Smaller lungs have smaller elastic recoil forces and hence a lower PTM. Air does not escape from the lungs into the pleural space. There is higher partial pressure of carbon dioxide in the blood than in the alveoli so carbon dioxide does not move from lungs to blood.

What happens to the size of veins emptying into the right side of the heart during forced exhalation? Assume pressure inside the veins stays constant. The veins increase in size. The veins decrease in size. The veins stay the same size.

The veins decrease in size. Explanation Transmural pressure (PTM = Pin - Pout) dictates the final volume of a flexible structure. In this case: Pin = Pressure inside the blood vessel Pout = Pressure outside the blood vessel (thoracic cavity) To forcefully exhale, one must build positive pressure in the thoracic cavity. This will increase Pout, thus decreasing PTM and decreasing the overall volume of the veins draining into the right side of the heart.

What happens to the size of veins emptying into the right side of the heart during inhalation? Assume that the pressure inside the veins remains constant. The veins increase in size. The veins decrease in size. The veins stay the same size.

The veins increase in size. Explanation Transmural pressure (PTM = Pin - Pout) dictates the final volume of a flexible structure. In this case: Pin = Pressure inside the blood vessel Pout = Pressure outside the blood vessel (thoracic cavity) To inhale, one must create negative pressure in the thoracic cavity. This will decrease Pout, thus increasing PTM and increasing the overall volume of the veins draining into the right side of the heart. The increase in the volume of the thoracic veins during inhalation allows more blood to flow into those veins from the abdominal veins.

Consider a patient with a systemic infection, experiencing septic shock, in whom the resistance of the systemic circulation of the cardiovascular system decreases significantly. Assume that the driving pressure (the ∆P of the system) stays the same. Why, under these circumstances, might you hear a cardiac murmur from blood flowing through one of the heart valves? (note: Murmurs are heart sounds related to vibrations of valve leaflets or vessel walls) The flow of blood across the valve is redu

The velocity of blood across the valve is increased. Explanation: Ohm's Law tells us that flow is equal to driving pressure divided by resistance. If resistance has decreased and driving pressure remains the same, then flow must increase. If the diameter of the valve has not changed, the increased flow across the valve also leads to an increase in the velocity of blood (remember, the units for flow and velocity are not the same; velocity = flow/cross sectional area of the valve opening). The increased velocity is likely to lead to turbulent flow according to Reynolds number. Turbulent flow causes increased vibration, which we hear as a murmur.

Blood is delivered to the right side of the heart via the vena cava, which is an easily compressed structure. If a patient comes to the emergency department with a pneumothorax, causing the pressure in their thorax to rise (i.e. a tension pneumothorax), what do you expect to happen to the size of the vena cava? The vena cava will increase in size. The vena cava will decrease in size. The vena cava will remain the same size. There is insufficient information to predict what will happen to the ve

The vena cava will decrease in size. Explanation The volume of flexible structures is determined by transmural pressure rather than the internal pressure of the structure. If the vena cava is easily compressed and the pressure outside the vena cava increases, then the transmural pressure of the vena cava will decrease. With a decreased transmural pressure, the size of the vena cava will decrease. This is one of the reasons that a tension pneumothorax is so dangerous, as a compressed vena cava reduces the amount of blood returning to the right side of the heart.

Cardiac tamponade is a disease process in which blood fills the sac around the heart called the pericardium, thereby increasing the pressure immediately outside the heart. How will this affect the volume of the ventricles (heart chambers) and systemic blood pressure? The ventricles will increase in volume and blood pressure will increase. The ventricles will decrease in volume and blood pressure will increase. The ventricles will increase in volume and blood pressure will decrease. The ventricl

The ventricles will decrease in volume and blood pressure will decrease. Explanation Transmural pressure (PTM = Pin - Pout) dictates the final volume of a flexible structure. In this case: Pin = Pressure inside the heart Pout = Pressure outside the heart If fluid collects around the heart, it can compress the heart, therefore decreasing the volume of the heart chamber (e.g. the ventricle), even though the pressure inside the heart may rise. If the volume of the ventricle decreases, less blood can be pumped out (i.e. decreased cardiac output). Because Blood Pressure = Cardiac Output × Systemic Vascular Resistance, blood pressure will be decreased.

A healthy man of average fitness is undergoing an exercise physiology test. When he is near his exercise limit, he begins to feel short of breath. What is happening in his tissues? Tissues are anaerobically generating carbon dioxide because he is limited by his cardiovascular function. Tissues are anaerobically generating carbon dioxide because he is limited by his pulmonary function. Tissues are anaerobically generating lactic acid because he is limited by his cardiovascular function. Tissues

Tissues are anaerobically generating lactic acid because he is limited by his cardiovascular function. Explanation When healthy individuals reach their exercise limit, they are typically limited by their cardiovascular function: the tissues are no longer able to efficiently utilize the oxygen that is being delivered or the cardiovascular system is not able to deliver as much oxygen as the tissues require. People rarely reach the limit of their pulmonary function, as the lungs have a large reserve capacity. Anaerobic metabolism produces lactic acid, not CO2. It is believed that the products of anaerobic metabolism activate receptors that cause the sensation of dyspnea (shortness of breath).

Why does blood pH decrease when carbohydrates (as opposed to proteins or fats) are the primary fuel source during exercise? Tissues using carbohydrates for energy release ketoacids into the blood as a byproduct. Tissues using carbohydrates for energy do not absorb as much CO2 from the blood. Tissues using carbohydrates for energy reduce total ventilation. Tissues using carbohydrates for energy produce more CO2.

Tissues using carbohydrates for energy produce more CO2. Explanation During exercise, the body uses carbohydrates to rapidly generate energy. As the metabolic fuel switches to carbohydrates, the respiratory quotient increases to nearly 1.0, meaning that more CO2 is produced per mL of oxygen consumed. The excess CO2 generated by the tissues is released into the blood, where it combines with water to form H2CO3, which dissociates to HCO3- and H+. As the level of H+ in the blood increases, the pH drops. Cells using carbohydrates as fuel do not consume HCO3- from the blood. Tissues do not absorb CO2 from the blood; instead, they release CO2 into the blood as a waste product. Ketoacids are not released when carbohydrates are available as a source of energy.

In a disease called diabetes insipidus, patients frequently have excessive thirst and consume large quantities of fluids, wiping out the urea solutes from the interstitium of the kidney. In such cases, what do you think happens to the concentration of solutes in the urine (i.e. urine concentration)? Urine concentration becomes very low. Urine output increases but stays at the same concentration. Urine output decreases but stays at the same concentration. Urine concentration goes up.

Urine concentration becomes very low. Explanation: Under normal circumstances, the body conserves water by reabsorbing water from the distal tubule as it traverses the portion of the kidney that has a very high concentration of solute in the interstitium surrounding the tubule. If this solute in the interstitium is removed, the osmotic forces necessary to cause movement of water out of the tubule will be weakened. Water will remain in the tubule and exit the body as dilute urine.

Fred takes a trip to La Paz, Bolivia. At almost 12,000 feet above sea level, the oxygen content of the air in La Paz is significantly lower than it is in his hometown of Boston. When he arrives, chemosensors in his blood vessels detect the drop in oxygen saturation of his blood, and send signals to his nervous system to increase his ventilation. Meanwhile, his body continues to produce similar amounts of carbon dioxide. The result is that his oxygen levels move closer to normal, but his PaCO2 de

Urine pH will be higher several days after his arrival. Explanation Fred's increased ventilation results in a respiratory alkalosis. To compensate, his kidneys secrete fewer protons into the urine and do not reabsorb as much bicarbonate to try and reduce his pH to be closer to normal. This process takes several days. Therefore,

The thermostat-furnace system in a house is one non-biologic example of a homeostatic mechanism. Say that Fred lives in a house. The house has a room, where Fred spends most of his time, and a garage, where Fred does not spend much time. For Fred to be comfortable, the temperature must be 65 degrees or higher. The furnace uses gas to produce heat, and there is a limited amount of gas that the furnace can use each day. The room and the garage can be heated independently of each other. On a partic

Use less gas to heat the garage. Explanation Fred's comfort is dependent on a temperature greater than 65 degrees. The problem is that there is not enough gas to heat both the room and the garage. Using extra gas, therefore, is not a viable solution, so the latter two solutions can be eliminated. Since the goal is to maximize Fred's comfort given a limited gas supply, heating the room becomes a higher priority than heating the garage, since Fred spends most of his time in the room. The answer is therefore to use less gas to heat the garage. As you will learn, the body uses similar compensatory mechanisms all of the time. In conditions where resources are limited or compensation is not sufficient, mechanisms exist that prioritize the physiologic functions that are essential for life.

In the following scenarios, a substance is added to an aqueous solution and affects the acid-base equilibrium of the solution. Which of the substances below (indicated by W, X, Y, or Z) is a base? (select two answers) W is added and the concentration of protons decreases from 0.04 μM to 0.03 μM. X is added and binds with protons to become XH+. Y is added to an aqueous system and dissociates into protons and Y-. Z is added and the pH goes from 7.5 to 7.4.

W is added and the concentration of protons decreases from 0.04 μM to 0.03 μM. X is added and binds with protons to become XH+. Explanation For our purposes, a substance is considered an acid when it donates protons (substance Y), and a base when it accepts protons (substance X). pH is defined as pH = -log[H+] As the concentration of protons increases, pH decreases. Thus, a substance that increases concentration of protons and decreases pH is an acid (substance Z), while a substance that decreases the concentration of protons and raises the pH is a base (substance W).

A patient has ingested a chemical that removes all the proteins from their bloodstream. What will happen to the movement of fluid between the compartments? Water will diffuse into the intravascular space from the interstitial space. Water will diffuse from the intravascular space into the interstitial space. Water will not move because there is no change in net force. Water will move from the intracellular to extracellular space.

Water will diffuse from the intravascular space into the interstitial space. Explanation: By removing protein from the vascular space, the balance of Starling forces between the vascular and interstitial compartments has been disrupted. The difference in hydrostatic pressure in favor of movement of water to the interstitial space dominates, and water moves into the interstitial space. Protein concentration (oncotic forces) are not involved in water balance between intracellular fluid and extracellular fluid.

Fred loves La Paz, Bolivia so much that he decides to stay there for a couple years. All of the following changes could be considered a compensatory mechanism in response to the chronically decreased oxygen content EXCEPT an increase in amount of circulating red blood cells. an increase in oxygen extraction by perfused cells. an increase in hemoglobin's affinity for oxygen. a decrease in heart rate.

a decrease in heart rate. Explanation All of the above (except for a decrease in heart rate) are compensatory mechanisms (in response to chronically low oxygen content) for people who are acclimatized to living at high altitudes. An increase in the number of red blood cells would increase the oxygen content when PaO2 is low; similarly, an increase in hemoglobin's ability to bind oxygen would increase O2 content and an increased ability to extract oxygen would facilitate maintaining aerobic metabolism in the tissues. As it turns out, heart rate and stroke volume decrease (decreasing energy needs of the heart) - but the major compensatory mechanisms are the other ones listed above.

Renal tubular acidoses are conditions (often inherited) in which impaired renal acid excretion develops in otherwise normally functioning kidneys, leading to a metabolic acidosis. All of the following are plausible mechanisms for renal tubular acidoses EXCEPT a decrease in carbonic anhydrase activity (enzyme that catalyzes CO2 + H2O ⇌ HCO3- + H+). a decrease in reabsorption of metabolic acids from the tubule into the bloodstream. a decrease in bicarbonate reabsorption from the tubule into the

a decrease in reabsorption of metabolic acids from the tubule into the bloodstream. Explanation Renal tubular acidoses (RTAs) cause a metabolic acidosis by impairing bicarbonate restoration or impairing acid elimination; note that decreasing carbonic anhydrase activity will impair both since carbonic acid must be converted to CO2 in order to diffuse into the renal tubular cell to facilitate acid secretion and bicarbonate reabsorption. The only scenario that could not lead to a metabolic acidosis is decreased reabsorption of metabolic acids (this would increase blood pH).

Which of the scenarios below will perturb "steady state" and lead to an increase in the concentration of substance X in the body? Assume substance X is eliminated passively at a constant rate by the kidney and is only produced by cells in the body (i.e. is not ingested). (select two answers) a decrease in the elimination of substance X a decrease in the production of substance X an increase in the elimination of substance X an increase in the production of substance X

a decrease in the elimination of substance X an increase in the production of substance X Explanation A decrease in elimination or an increase in production of a substance in the body will increase its concentration in the body. On the other hand, a decrease in production or an increase in elimination of a substance in the body will decrease its concentration in the body.

A metabolic acidosis occurs when the amount of acids ingested and/or metabolic acids produced exceeds the ability of the kidneys to eliminate those acids. Certain causes of metabolic acidosis are associated with an increase in the anion gap. Complete the following statement: The increase in anion gap resulting from a metabolic acidosis occurs because of ____________ in the concentration of ____________ used to calculate the anion gap. an increase; cations an increase; anions a decrease; anions

a decrease; anions Explanation The anion gap is calculated by subtracting the serum bicarbonate levels from the sum of serum sodium and potassium levels: Anion gap = ([Na+] + [K+]) - ([Cl-] + [HCO3-]) An increased anion gap could therefore theoretically result from an increase in the measured cation levels or a decrease in the measured anion levels. Adding a metabolic acid acid (e.g., lactic acid) to the system can increase the anion gap because the acid dissociates into its conjugate base (lactate) and a proton. Bicarbonate in the blood binds this proton and is converted to CO2, which can be breathed out of the body by the lungs, resulting in the net loss of bicarbonate, a measured anion. The negatively charged lactate, which is not measured for the calculation of the anion gap, remains in the place of the bicarbonate that was consumed, meaning there was a net decrease of measured anions (recall that net electrical neutrality is conserved as one anion replaces another). Note: serum p

All of the following reactions produce metabolic acids that are eliminated by the kidney or metabolized by the liver, EXCEPT aerobic metabolism of carbohydrates to produce carbon dioxide. breakdown of proteins to produce phosphoric and sulfuric acid. anaerobic metabolism of carbohydrates to produce lactic acid. lipid (fat) metabolism to produce fatty acids and ketones.

aerobic metabolism of carbohydrates to produce carbon dioxide. Explanation Respiratory acids are the product of the aerobic metabolism of carbohydrates, breathed off by the lungs as CO2, while metabolic acids are generally the product of (1) anaerobic metabolism of carbohydrates and (2) metabolism of other biomolecules (proteins, fat, nucleic acids) and are eliminated by the kidney (and/or metabolized by the liver).

Basic functions of the kidneys include: conserving water filtering toxins or byproducts of metabolism getting rid of excess dietary fluid and salt all of the above

all of the above Explanation: All of the above. We rely on the kidney to filter byproducts of metabolism, as well as to filter tremendous amounts of water, reclaiming most of it but selectively fine tuning the urine output to get rid of excess salt and water. The kidney must also be able to conserve water, given that we must maintain a total body water of roughly 60% by weight, while existing in a dry environment.

Strategies to increase airflow during inspiration (while breathing in) through a narrowed airway in the lungs may include: decreasing the alveolar pressure upon inspiration to a greater extent by breathing more deeply increasing the pressure at the mouth giving a bronchodilator to decrease the resistance of the airway all of the above

all of the above Explanation: Ohm's Law tells us that flow is equal to driving pressure divided by resistance. To increase flow, one has to increase driving pressure by altering the pressure on either end of the tube (in this case, the mouth or the alveolus), or reduce the resistance. A bronchodilator relaxes smooth muscle surrounding the medium and large diameter airways, increases airway radius, and thereby reduces resistance.

Regular exercise can increase an individual's "aerobic capacity" (which is a function of oxygen delivery and how efficiently oxygen is utilized by metabolically active tissues) in several ways. All of the following are plausible mechanisms by which regular exercise increases aerobic capacity EXCEPT an increase in the mitochondrial density in muscle cells. an increase in the density of capillaries in muscle tissue. an increase in the systemic vascular resistance (SVR). an increase in the heart's

an increase in the systemic vascular resistance (SVR). Explanation Note that an increase in stroke volume leads to an increase in cardiac output and thus, oxygen delivery. Moreover, an increase in mitochondrial density in muscle cells allows metabolically active muscle to more efficiently utilize oxygen. Finally, an increase in capillary density in muscle cells leads to improved diffusion of oxygen into muscle cells. While SVR decreases during an exercise session, at rest, SVR would not differ substantially between trained and untrained individuals.

You see a patient in the hospital with a rapid heart rate. You suspect that she had poor oxygen delivery to her tissues and her body compensated for this by increasing her cardiac output. What might have caused her poor oxygen delivery? decreased oxygen consumption by the tissues breathing 100% oxygen from a mask decreased CO2 production anemia (low hemoglobin)

anemia (low hemoglobin) Explanation Recall that oxygen delivery depends on cardiac output and the oxygen content of the blood (O2 delivery = cardiac output × O2 content). Oxygen content depends on the amount of oxygen in the blood bound to hemoglobin and the amount of oxygen dissolved in the blood (the former greatly exceeds the latter). If the amount of hemoglobin in the blood is reduced, then there will be less oxygen bound to hemoglobin in the blood, and the blood will have decreased oxygen content. To compensate, the cardiovascular system must increase the cardiac output. Decreased oxygen consumption by the tissues would not cause reduced oxygen delivery, as more oxygen would remain in the venous system after circulating through the tissues. Breathing 100% oxygen would increase oxygen content by increasing oxygen entering the lungs. Decreased CO2 production would not negatively affect oxygen delivery.

You have heard about the thermostat-furnace system in a house as a non-biologic example of a homeostatic mechanism. In the human body, structures called baroreceptors monitor the pressure in blood vessels to try and maintain it in a safe range. If blood pressure drops, the baroreceptors send signals that cause the heart to increase contractility (pump harder) in an effort to restore the pressure. Which of the answers below correctly matches each component of the human body example with correspon

baroreceptor - thermostat; heart - furnace; blood pressure - temperature Explanation Both systems have a sensor that senses changes in some variable. In the house, the variable is temperature, while in the body, the variable is blood pressure. The sensor of the house is the thermostat, while in the body, it is the baroreceptor. In the house, the furnace compensates for changes in temperature (by producing heat), while in the body, the heart compensates for changes in blood pressure (by increasing contractility). Changing contractility is a mechanism by which the body tries to move back to the "set point" of blood pressure, but contractility is not the variable sensed by the baroreceptors.

A 22-year-old patient arrives in your clinic by ambulance. The emergency medical technicians tell you that his friends found him unresponsive with a needle in his arm and called for an ambulance. He is found to have slow, shallow breathing and has an arterial pCO2 of 80 (normal = 40) and pH of 7.08 (normal = 7.40). A member of the ambulance team sticks around for a while and asks you why you didn't give the patient HCO3- to resolve the patient's low pH. You tell her that the patient's acidosis

bicarbonate administration would only increase the already high pCO2. Explanation Respiratory acidosis is caused by the production of CO2 without adequate removal. When there is too much CO2, the following reaction is driven to the right: CO2 + H2O ⇌ H2CO3 ⇌ HCO3- + H+ producing H+ and HCO3-. If HCO3- is administered, the reaction will be driven back to the left, but this will only result in increased production of CO2, which does not address the problem of high CO2 levels in the blood (note: though pH may normalize temporarily with addition of bicarbonate, excess CO2 levels in the blood are still problematic). This is why HCO3- is not an adequate "buffer" for a respiratory acidosis. Injection of bicarbonate would produce a metabolic alkalosis and a rise in the pH, which is not present in this case.

A biological system is in steady state. Now the input to that system increases. How might the system respond to achieve a new steady state? by holding elimination constant by holding production constant by decreasing elimination by increasing elimination

by increasing elimination Explanation When the input to a system in steady state increases and the other variables do not change, the system will enter dynamic conditions as the level of the substance increases. Recall that Input + Production = Elimination. In order to achieve a new steady state, elimination must increase or production must decrease (or both). Biologically, the most common scenario is that elimination increases as the concentration of the substance increases.

Epinephrine (adrenaline) has been called the "fight or flight" hormone. It induces physiological changes in the cardiovascular system that help prepare the body for the increased demand of intense activity. What is the likely effect of epinephrine on cardiac output? cardiac output remains the same cardiac output decreases cardiac output increases

cardiac output increases Explanation Epinephrine increases the supply of oxygen to tissues to handle increased demand. It does so, in part, by increasing cardiac output by enhancing strength of contraction and increasing heart rate. Epinephrine has complex effects on systemic vascular resistance (SVR) that depend on dosage and individual differences.

During normal inhalation, the diaphragm and muscles of the chest wall work to produce a negative pressure in the thorax, leading to an influx of air into the lungs. During exhalation, the muscles relax, and the pressure in the thorax becomes less negative, allowing the elastic recoil of the lungs to push air out. A patient with paralyzed respiratory muscles cannot generate the negative thoracic pressure necessary for inhalation. An external device called a mechanical ventilator can be used to e

decrease; expiratory Explanation With positive pressure ventilation, the transmural pressure of the lungs is increased by increasing the pressure of the air filling the lungs. Intrathoracic pressure is positive (it increases) during inhalation and approaches zero (decreases, but remains positive) during exhalation. The pressure in the thorax is greater overall throughout the respiratory cycle than in a normally respiring patient. The heart resides in the thorax. Higher pressures in the thorax will increase Pout for the heart, thereby decreasing transmural pressure and heart volume. As we learned previously, in an otherwise healthy heart with all else being equal, greater heart volumes lead to bigger cardiac outputs, while lower heart volumes lead to smaller cardiac outputs. Therefore, a higher pressure in the thorax leads to a decrease in cardiac output. As the positive pressure is applied during inhalation, and reduced during exhalation, the pressure of the thorax will be greatest du

A person with morbid obesity who is carrying extra weight in the abdominal area complains of shortness of breath when lying flat (face up), with difficulty breathing in. Consider the pressure change needed to cause a given change in volume for the chest, and note that the abdomen is considered to be the lower border of the chest wall. When lying down, the compliance of the chest wall in this morbidly obese person is: increased from normal because there needs to be a greater change in pressure i

decreased from normal because there needs to be a greater change in pressure in the lungs to raise the abdomen Explanation The abdomen is often viewed as the inferior portion of the chest wall because the diaphragm, as it descends, must displace the abdominal contents. When you lie flat, the pressure that the abdomen exerts on the diaphragm is greater; more work must be done for any given volume of air to be moved into the lungs, and the compliance is reduced relative to the upright position. In the case of the morbidly obese person described in this question, the extra adipose tissue increases the pressure in the abdomen enough when lying down to cause shortness of breath. Remember that delta P is in the denominator of compliance, so that a greater change in pressure needed to produce the same change in volume is indicative of decreased compliance.

You learned that baroreceptors activate the sympathetic nervous system in response to a low blood pressure. The baroreceptor in the carotid arch also responds to high blood pressure by activating the parasympathetic nervous system. Given what you know about blood pressure and homeostasis, what might be the effects of parasympathetic activation by baroreceptors? decreased heart rate, constricted peripheral blood vessels increased heart rate, constricted peripheral blood vessels decreased heart r

decreased heart rate, dilated peripheral blood vessels Explanation When blood pressure is too high, the baroreceptor activates the parasympathetic nervous system to decrease blood pressure. Recall that: Blood pressure = ΔP = MAP - CVP = Q x SVR, where Q = cardiac output, SVR = systemic vascular resistance, MAP = mean arterial pressure, and CVP = central venous pressure. Cardiac output = Q = HR x SV, where HR = heart rate and SV = stroke volume. To lower blood pressure, systemic vascular resistance (SVR) and/or cardiac output (Q) must be decreased. SVR can be decreased by dilating peripheral blood vessels, and Q can be decreased by decreasing heart rate. Indeed, the parasympathetic nervous system responds to increased blood pressure, in part, by decreasing heart rate and dilating peripheral blood vessels.

Robert is a 60-year old patient with heart failure. As such, his heart doesn't pump blood well, and he has expanded blood volume and elevated venous pressures. Robert asks his doctor why he has so much swelling in his legs (increased interstitial water). His diet has been good; he makes sure he gets adequate protein. Robert's doctor explains that water and sodium have increased Robert's blood volume. Based on this, you think that inside Robert's leg veins, there is: increased oncotic pressure a

decreased oncotic pressure and increased hydrostatic pressure Explanation: As salt and water are absorbed from the intestines into the bloodstream, the hydrostatic pressure in the vascular compartment increases. This results in movement of salt and water into the interstitial space. This process is further enhanced by the dilution of the protein in the vascular compartment, which reduces the oncotic pressure. The balance of hydrostatic and oncotic pressures between the vascular and interstitial compartments has been disrupted. Water leaves the vascular space until the two compartments are in equilibrium.

A patient has ankylosing spondylitis, a condition that causes the joints of the vertebral column and the joints connecting the ribs to the vertebral column to become stiff. The patient complains of shortness of breath. How do you predict his chest wall compliance has changed over time? increased, because a greater change in pressure is needed to expand the chest wall in normal breathing decreased, because a greater change in pressure is needed to expand the chest wall in normal breathing increa

decreased, because a greater change in pressure is needed to expand the chest wall in normal breathing Explanation In ankylosing spondylitis, the chest wall is stiffer and therefore requires greater force (and hence higher pressure) to expand. A greater change in pressure is needed to produce the same change in volume of the chest wall. Given that change in pressure (delta P) is in the denominator of the equation for compliance, this finding signifies a decrease in compliance in patients with this disease.

The compliance of a flexible structure, such as a balloon, is determined by a curve (volume vs. pressure), rather than being constant. As you inflate a balloon, it gets more and more difficult to achieve a given change in volume. Stated another way, you have to supply a greater change in pressure to achieve a given change in volume when the balloon is at higher volumes vs. when it is at lower volumes. This means that the compliance of the balloon: increases at lower volumes then decreases at hi

decreases as you inflate it Explanation If you graph the relationship between the pressure and the volume of the balloon, with pressure on the horizontal axis and volume on the vertical axis, the compliance is the slope of the pressure-volume curve. For elastic structures, typically the compliance decreases as the volume increases (in other words, the curve flattens out).

For a spherical structure with an air-liquid interface, such as a soap bubble, surface tension contributes to pressure inside by: exerting forces that, for a given surface molecule, point inward exerting forces that, for a given surface molecule, point outward exerting forces that hold the bubble open (keep it from collapsing) exerting forces that act to maximize the surface area of the bubble

exerting forces that, for a given surface molecule, point inward Explanation For a soap bubble, surface tension forces between neighboring surface molecules act to collapse the bubble and tend to minimize its surface area (hence, the spherical shape). The surface tension forces on a given molecule, when summed, point toward the center of the bubble. The pressure inside the bubble is responsible for the force counteracting the surface tension and keeping the bubble from collapsing.

The intravascular and interstitial compartments are together referred to as the ________ space.

extracellular

A healthy adult can increase his "aerobic capacity" or anaerobic threshold (AT) by increasing his maximal ventilation. true false

false Explanation The AT is determined primarily by (1) oxygen delivery and (2) ability of tissue to extract oxygen from the bloodstream. Since the cardiovascular system limits exercise capacity in healthy individuals due to limits in delivering and utilizing oxygen, increased ventilation is unlikely to have an effect on aerobic capacity in a healthy adult.

When measuring vital signs, we always measure temperature, heart rate, blood pressure, and respiratory rate. The human body has specialized receptors that sense these values, either directly or indirectly. This allows the body to make adjustments when necessary to try to maintain normal function. Which of the following receptor types would be the least necessary for the body to measure the four vital signs? baroreceptors (sense pressure) thermoreceptors (sense temperature) fat receptors (sense

fat receptors (sense cell fat concentration) Explanation The human body relies on the function of baroreceptors, thermoreceptors, and chemoreceptors to maintain appropriate blood pressure, core body temperature, and blood pH. Chemoreceptors measure pH, pCO2, and pO2, which are intimately connected with respiratory rate. There are no fat receptors in the body and the concentration of fat is less important when thinking about these four vital signs that implicate major organ systems.

As fluid moves along normal systemic capillaries, Starling forces favor ____________ at the arterial ends of capillaries and ____________ at the venous ends of capillaries. no movement of fluid; no movement of fluid filtration; no movement of fluid reabsorption; filtration filtration; reabsorption

filtration; reabsorption Explanation: The Starling forces at the arterial end of a typical systemic capillary favors filtration, given that the net hydrostatic pressure pushing fluid into the interstitium outweighs the net oncotic pressure pulling fluid into the capillary. At the venous end, the opposite is true; reabsorption is favored because the hydrostatic pressure inside the capillary has decreased such that the net oncotic pressure pulling fluid into the capillary outweighs the net hydrostatic pressure pushing fluid out. The amount of fluid filtered per unit time at the arterial end of a systemic capillary and reabsorbed at the venous end is typically well under 1% of the total flow through the capillary.

Increases in SVR can be due to (select 3 answers) elevated central venous pressure (CVP). fixed blockages of blood vessels. constriction of blood vessels. elevated angiotensin levels.

fixed blockages of blood vessels. constriction of blood vessels. elevated angiotensin levels. Explanation Systemic vascular resistance is a measure of the resistance of the systemic circulation as a whole. As such, anything that narrows blood vessels has the potential to increase SVR if there is not dilation of vessels elsewhere. Angiotensin is a vasoconstrictor and would lead to narrowing of vessels.

You have two balloons. The red balloon is stiffer than the blue balloon, meaning it has higher elastic recoil forces at a given volume. You inflate them both to the same volume. Select the answer that correctly completes the following statements: The pressure inside the red balloon will be ____________ than the pressure inside the blue balloon. The transmural pressure of the red balloon will be ____________ than the transmural pressure of the blue balloon. greater, greater greater, less less, g

greater, greater Explanation To inflate a balloon, sufficient pressure must be applied to overcome the elastic recoil of the balloon. The higher elastic recoil of the red balloon contributes to a higher pressure inside the red balloon than the blue balloon at a given volume. Since transmural pressure is defined as PTM = Pin - Pout and Pout(red) = Pout(blue) = Patm and Pin(red) > Pin(blue) PTM of the red balloon will be greater than PTM for the blue balloon at a given volume.

A 26-year-old man is involved in an automobile accident. He arrives to the emergency department with a blood pressure of 122/73 (normal = 90-120/60-80). On rapid assessment of the patient, you suspect that that he has abdominal bleeding. You are surprised that his blood pressure is within the normal range. You would expect to find that his _____________ is _____________, allowing him to maintain his blood pressure. heart rate; increased heart rate; decreased respiratory rate; increased respirat

heart rate; increased Explanation In this case, the heart rate is most likely elevated as a compensation to maintain the blood pressure, as MAP - CVP = Q x SVR, and Q = HR x SV (MAP = mean arterial pressure, CVP = central venous pressure, Q = Cardiac output, SVR = systemic vascular resistance, HR = heart rate, and SV = stroke volume). Note that the patient may be breathing rapidly due to pain, but this would not serve to maintain his blood pressure.

Your patient has a narrowed airway (trachea) and is working very hard to breathe. Which of the following gases (listed below) would improve airflow through this narrowing? The densities of the gases are (in kg/m^3): Argon = 1.661; Nitrogen = 1.165; Helium = 0.1664; Oxygen = 1.331. Assume the viscosities of the gases are approximately the same. Assume in each case that the mixture of gas inhaled is 21% oxygen with the remainder comprising the other gas noted. 100% oxygen argon plus oxygen nitrog

helium plus oxygen Explanation: As an airway narrows, flow decreases as resistance increases, given the same driving pressure. If one reaches a critical point, as determined by the Reynolds number, and flow changes from laminar to turbulent, resistance rises significantly. A small Reynolds number is more likely to have laminar than turbulent flow. Reynolds number is proportional to the density of the fluid flowing through the tube. Thus, the gas with the lowest density is most likely to be associated with laminar flow. The mixture of helium and oxygen provides the lowest density gas. In fact, this mixture is called heliox and is sometimes used to reduce the work of breathing in medical emergencies like the one described above.

Predict the values you would see in a person who is exercising but cannot increase their ventilation. low PCO2, low pH low PCO2, high pH high PCO2, low pH high PCO2, high pH

high PCO2, low pH Explanation When we exercise, our respiratory quotient (R) increases because we are burning more carbohydrates. Moreover, higher metabolic demands lead to more CO2 being generated during exercise. If ventilation does not increase under these circumstances, PCO2 will rise. Since CO2 rapidly combines with water to become H2CO3, and subsequently H+ and HCO3-, the blood becomes more acidic (pH decreases).

A 49-year-old woman was rescued from a house fire. The firefighters report that she was rescued from a smoke-filled room. You know that oxygen is consumed and carbon dioxide produced during combustion. When looking at the patient's blood work, what would you expect to find? (select all correct answers) high pCO2, high serum HCO3- levels high pCO2, low serum HCO3- levels high pCO2, high serum H+ levels high pCO2, low serum H+ levels

high pCO2, high serum HCO3- levels high pCO2, high serum H+ levels Explanation For an acidosis caused by high levels of CO2 in the blood, hemoglobin acts as buffer by binding protons (H+). Consequently, when H+ remains trapped in the red blood cell bound to hemoglobin, HCO3- is released into the bloodstream. Therefore, in an instance where a patient has respiratory acidosis (acidosis caused by high levels of CO2), you would expect to see high levels of CO2 and high levels of HCO3-. While buffering mitigates the impact of the respiratory acidosis, the concentration of protons does increase as carbonic acid forms from the combination of CO2 and water.

Water moves from areas of ______ hydrostatic pressure to areas of ______ hydrostatic pressure. high, low low, high

high, low Explanation: Forces resulting from hydrostatic pressure result in movement of water from areas of high hydrostatic pressure to areas of low hydrostatic pressure.

What are two ways to increase the volume of a balloon? (select two answers) increase Pin and Pout the same amount hold Pin constant and decrease Pout increase Pin and hold Pout constant hold Pin constant and increase Pout decrease Pin and increase Pout

hold Pin constant and decrease Pout increase Pin and hold Pout constant Explanation Volume is not dependent solely on the pressure inside a flexible structure; it is dependent on transmural pressure, or the difference between the pressure inside and pressure outside (PTM = Pin - Pout). In order to increase volume, you have to increase the transmural pressure. Either (1) increasing Pin and holding Pout constant or (2) holding Pin constant and decreasing Pout will both cause a net increase in transmural pressure, thereby increasing the volume of the balloon. There are other scenarios that can increase volume (e.g. decreasing Pout and increasing Pin), but these are the only two appropriate answers in this list.

Imagine a system that is in a steady state with respect to a substance. The substance is eliminated at a rate proportional to its concentration in the blood. Which of the following might happen to the amount of the substance when the production of this substance increases? Assume you start measuring when production increases and that there is no input of this substance into the system. The amount of the substance will decrease, then level off. increase, then level off. decrease indefinitely. in

increase, then level off. Explanation A system is in a steady state when the input to the system, plus what the system creates, equals what is eliminated from the system. If we increase the production of a substance, the system will no longer be in steady state, and the amount of the substance will begin to increase. Since the rate of elimination increases proportionately to the concentration of the substance in the blood, the concentration will continue to increase until the rate of elimination is again equal to the new rate of production and a new steady-state has been achieved.

A patient overdoses on opioid pain medications, which causes central respiratory depression in the brainstem, which results in slow, shallow breathing. The bicarbonate level in the blood will ____________ and the carbon dioxide level in the blood will ____________. increase; increase increase; decrease decrease; increase decrease; decrease

increase; increase Explanation Slow, shallow breathing would impair elimination of carbon dioxide from the lungs, resulting in a respiratory acidosis. In a respiratory acidosis, the carbon dioxide levels rise. Immediate buffering of protons occurs in red blood cells, producing bicarbonate; over several days, the kidneys also eliminate protons and increase bicarbonate stores. Note that the bicarbonate (conjugate base) changes in the same direction as carbon dioxide (conjugate acid).

A 66-year-old man visits you in the outpatient renal clinic. He has significant renal dysfunction and his urine output has significantly diminished over the past several days. He says that he continues to consume his normal diet and drink water at his normal rate. He reports no other losses of water, such as vomiting, diarrhea, or excessive sweating. What would you expect to have happened with regards to his weight? increased decreased stayed the same not enough information

increased Explanation In a steady state situation, input = output. If something changes to either of these factors, then the system will no longer be in steady state. If the output of a substance decreases without any change in its input, this will result in an increase in the amount of the substance. In this case, the patient's fluid output has decreased, so the amount of fluid in his body will increase, causing his weight to rise.

In emphysema, the elastic fibers of the lungs are injured and elastic recoil is greatly reduced. As such, the compliance of the lungs is: increased decreased unchanged

increased Explanation When stretched upon inflation, elastic fibers contribute to the elastic recoil of the lungs by exerting forces that return the lungs to their resting position. When elastic fibers are injured, the lungs have reduced elastic recoil and, as is the case with an stretchy balloon, less of a change in pressure is needed to cause a given change in volume. Hence the compliance is increased. Unfortunately, in emphysema this causes other problems, given that elastic recoil contributes to breathing out and elastic fibers also help hold airways open via radial traction forces.

Hyperventilation is defined as a level of ventilation (breathing) that results in a decrease in serum CO2 to below normal levels. Which of the following might you expect as a result of hyperventilation? increased concentration of bicarbonate in the blood increased protonation of hemoglobin proteins increased blood pH respiratory acidosis

increased blood pH Explanation CO2 in the blood is in equilibrium with carbonic acid: CO2 + H2O ⇌ H2CO3 ⇌ HCO3- + H+ This reaction is catalyzed in red blood cells, where anionic proteins like hemoglobin serve to buffer the system by binding some of the protons produced, while the resultant bicarbonate diffuses back out of the cell. A decrease in CO2 in the serum will therefore lead to a decrease in the protonation of hemoglobin proteins, a decrease in serum bicarbonate levels, and a respiratory alkalosis, which results in an increase in blood pH.

Which two of the following are results attainable from training that MOST contribute to increased performance in exercise? (select two answers) increased efficiency of aerobic metabolism of carbohydrates increased minute ventilation at a given V02 increased stroke volume of the heart increased lung capacity

increased efficiency of aerobic metabolism of carbohydrates increased stroke volume of the heart Explanation Training enhances oxygen utilization in tissues and the heart's ability to pump blood and deliver oxygen to the tissues. The factor that limits exercise in a normal individual is thought to be the ability to deliver and utilize oxygen, not lung capacity—increased lung capacity would therefore not be as significant a factor. Increased minute ventilation at a given VO2 might suggest a heavier reliance on anaerobic metabolism, which is a less efficient source of energy than aerobic metabolism. Because aerobic metabolism is a more efficient source of energy, an increase in the body's ability to extend aerobic metabolism to supply the body's energy demands would tend to increase a person's ability to push himself or herself in exercise. An increased stroke volume of the heart would contribute to higher cardiac output, which would lead to better oxygen delivery.

Which of the following would NOT decrease venous oxygen content? increased oxygen consumption in the tissue decreased oxygen content in arteries increased mean arterial pressure decreased cardiac output

increased mean arterial pressure Explanation Increased oxygen consumption in the tissue leads to increased extraction of oxygen from the blood, thus reducing venous oxygen content. Oxygen consumption can remain constant even if arterial oxygen content or cardiac output decrease; such decreases are offset by increased oxygen extraction from the blood, leading to reduced venous oxygen content. Increased MAP by itself is not associated with decreased venous oxygen content.

As a person is climbing a flight of stairs, you notice that his ventilation has increased. You suspect that he is trying to increase the amount of oxygen being brought into his body. Increased ventilation during physical activity correlates with increased oxygen consumption by tissues. increased arterial oxygen content. increased venous oxygen content. increased CO2 dissolved in blood.

increased oxygen consumption by tissues. Explanation There is a positive correlation between oxygen consumption at the level of the tissues and ventilation. Increased venous oxygen content would indicate that less oxygen is being utilized by tissues, which would instead correlate with reduced ventilation. During submaximal exercise (such as stair climbing), arterial oxygen content and CO2 dissolved in blood don't change to any significant degree. The exact cause of increased ventilation during submaximal exercise is not known.

Erythropoietin (EPO) is a hormone that functions to stimulate the production of red blood cells. EPO has a more infamous history as an illegal "blood-doping" agent taken by athletes to increase their aerobic capacity. Which physiological change best explains how EPO can increase an athlete's capacity for aerobic exercise? increased muscle cell extraction of oxygen from the bloodstream increased oxygen content increased ventilation rate increased stroke volume

increased oxygen content Explanation Aerobic exercise is limited by O2 delivery to the tissues AND the ability of the tissues to extract the delivered O2. Recall that: O2 delivery = Cardiac output × O2 content Thus, increasing cardiac output (equal to stroke volume × heart rate) and/or O2 content (equal to dissolved O2 + O2 bound to hemoglobin) will increase O2 delivery and overall aerobic capacity for exercise. In this case, EPO causes an increase in the number of red blood cells, increasing hemoglobin concentration, and thus increasing O2 content. Note that increasing stroke volume and increasing extraction of O2 from the blood also increase aerobic capacity, but are not effects of EPO. EPO's effect on aerobic capacity is not due to effects on ventilation rate during exercise.

The hormone angiotensin is important for regulating the tone (extent of constriction) of blood vessels. Specifically, angiotensin is a vasoconstrictor (blood vessels become narrower in response to angiotensin). Suppose an individual's angiotensin level increases, but their cardiac output remains the same. Which of the following physiological changes are likely to be present? (select two answers) increased systemic vascular resistance (SVR) decreased systemic vascular resistance (SVR) increased

increased systemic vascular resistance (SVR) increased mean arterial pressure (MAP) Explanation Angiotensin causes vasoconstriction, which increases SVR. If the heart maintains normal cardiac output, you can deduce from the P=Q*SVR equation that P must increase. MAP is the major contributor to P, and it is therefore likely that MAP has increased. Venous pressure (in this case, central venous pressure or CVP) is less affected because angiotensin's effect on the veins is less than its effect on the arteries. Angiotensin is one of the key hormones that regulates blood pressure.

Respiratory dysfunction can lead to blood pH that is higher or lower than usual. Which of the following might you expect to lead to high blood pH (alkalosis)? repeatedly inhaling and exhaling into a paper bag increased ventilation beyond metabolic needs a severe asthma attack a 400 meter walk

increased ventilation beyond metabolic needs Explanation Activities that lead to a higher concentration of CO2 in the blood lead to lower pH, as the gas dissolves in blood and combines with water to form carbonic acid, which dissociates into bicarbonate and a proton. Activities that lead to decreased CO2 levels will lead to increased pH, as carbonic acid forms CO2 and H2O. Breathing into a paper bag functions to concentrate the CO2 of inhaled air, resulting in a decrease in blood pH. Increasing ventilation beyond metabolic needs, on the other hand, leads to increased expulsion of CO2 beyond what is produced, which shifts the bicarbonate buffer system of the blood to consume bicarbonate and hydrogen ions, thus raising pH. During a severe asthma attack, the airways are constricted, limiting ventilation and leading to increased CO2 levels. In a 400m walk, ventilation will increase in concert with increased CO2 production and pH will not change.

In a process called vascular autoregulation, blood flow through a vessel is maintained (through intrinsic mechanisms) nearly constant over a range of driving pressure. In this range, what happens to the vessel resistance as driving pressure (∆P) increases? stays the same increases linearly in proportion to the change in driving pressure increases non-linearly in proportion to the change in driving pressure decreases in proportion to the change in driving pressure

increases linearly in proportion to the change in driving pressure Explanation: According to Ohm's Law, flow is determined by the driving pressure divided by the resistance. The concept of autoregulation is that flow is constant over a wide range of driving pressure. Thus, resistance of the vessels must be changing to compensate for the alterations in driving pressure (i.e., blood pressure in human physiology) to maintain constant flow. Note that the autoregulation phenomenon is significant in a few organs (primarily brain and kidney), and it is only able to compensate for changes in blood pressure within a particular range of pressures.

A man driving in a snowstorm loses control of his car and crashes into a tree. Among other injuries, he suffers a laceration of a major artery. By the time the paramedics get to him, his blood pressure has dropped to dangerous levels. Of the options below, which two organs would you expect to lose perfusion the fastest? intestines and heart intestines and skin brain and heart brain and skin heart and skin

intestines and skin Explanation An important concept in homeostasis is that of prioritization. If the heart were to cease pumping blood, the rest of the body would have no blood circulation, and if the brain were not functioning, there would be no central controller for the rest of the organs. A loss of perfusion of these organs would therefore have the most immediate catastrophic effects as compared to a loss of perfusion in other organs, like the intestines and skin. It makes sense, then, that the body has evolved to respond to a drop in blood pressure by prioritizing perfusion of the brain and heart.

Imagine a medication that would completely eliminate surface tension in the lungs. Compliance of the lungs would become: larger than normal same as normal less than normal

larger than normal Explanation Surface forces as well as elastic forces both contribute to the recoil forces in the lung. If the surface forces are completely eliminated due to the medication, then recoil forces will decrease, and less of a change in pressure will be needed to cause a given change in volume. In other words, the compliance will be larger than normal.

Consider what you have learned about homeostasis and acid-base equilibrium to predict how the body will respond to changes in pH. For each compensatory mechanism listed below, indicate the stimulus that is most likely to lead to its activation. Compensatory mechanism Stimulus likely to activate compensatory mechanism Increased ventilation Select an option high pH low pH Decreased ventilation Select an option high pH low pH Increased renal bicarbonate generation Select an option high pH low

low pH high pH low pH high pH Explanation Acidemia is an increase in the hydrogen ion concentration in the blood above the normal range (corresponding to a decrease in pH). The lungs and kidneys both respond in an effort to correct the acidemia. Ventilation will increase to breathe off additional CO2 and reduce the amount of acid produced from the conversion of CO2 to H2CO3 (carbonic acid) in the blood. The kidneys will increase new bicarbonate generation in order to increase the amount of base in the blood to counteract the acid. The opposite is true for an alkalemia - ventilation levels should fall in order to increase CO2 levels in the blood and bicarbonate reabsorption from the urine should decrease. The correct answers are indicated above.

Water moves from areas of _____ solute concentration to areas of _____ solute concentration. high, low low, high

low, high Explanation: Osmotic forces cause water to move to equilibrate solute concentration across a semi-permeable membrane, such as the cell wall. Water will therefore move from areas of low solute concentration to areas of high solute concentration.

Carbonic anhydrase is an enzyme that catalyzes the interconversion of carbon dioxide and water to bicarbonate and a proton: CO2 + H2O ⇌ HCO3- + H+ This enzyme is necessary for the reaction shown above to occur rapidly enough for normal kidney function. Which of the following might you expect to occur if we inhibited the carbonic anhydrase present in renal tubular cells? increased excretion of protons into the lumen of the renal tubule increased secretion of bicarbonate into the blood respirat

metabolic acidosis Explanation The ability of the kidney to eliminate metabolic acids depends on the following steps: In the blood streamA metabolic acid donates a proton to bicarbonate to form carbonic acid.The carbonic acid is converted to CO2.The CO2 diffuses into the renal tubular cell. In the renal tubular cell The CO2 is converted back into bicarbonate and a proton (catalyzed by carbonic anhydrase). The proton is excreted out of the cell and into the lumen of the renal tubule, ultimately to leave the body via the urine. The bicarbonate is secreted back into the bloodstream. The inhibition of carbonic anhydrase in renal tubular cells would result in the cells' inability to catalyze the conversion of CO2 back into bicarbonate and a proton. It would therefore reduce the excretion of protons by the kidney, and reduce the amount of bicarbonate secreted back into the bloodstream, and a metabolic acidosis would result.

The purpose of a physiologic buffer is to (select two answers) transform an added acid and/or base into physiologically useful product(s). keep the concentration of the conjugate acid and conjugate base constant. mitigate the change in pH in response to an added acid and/or base. facilitate proper function of enzymes and metabolic processes.

mitigate the change in pH in response to an added acid and/or base. facilitate proper function of enzymes and metabolic processes. Explanation A physiological buffer mitigates the impact of any added acid or base to a system's overall pH (in medicine, we are usually referring to the pH of the blood). But why do we have buffers (recall that a buffer usually consists of a weak acid and weak base pair; the most well-known buffer in the body is the CO2 and HCO3- system)? One reason is to allow for the proper functioning of enzymes, which can only function optimally within a narrow pH range. Note: the CO2 and HCO3- buffer can mitigate against changes in pH well for metabolic acidosis (in which there is an external "non-carbonic" conjugate acid formed), but it does NOT effectively buffer against respiratory acidosis (in which there is perturbation of the CO2 and HCO3- equilibrium) because the removal of a proton leads to the formation of CO2, which is already elevated in a respiratory acido

Osmotic forces related to protein concentration differences are referred to as ________ forces.

oncotic

Bob and Bill are 70-year old twins. Bob has a narrowing in one of his major leg arteries while Bill's arteries are normal. They walk together at a normal pace and Bob begins to have cramping leg pain in his affected leg due to the narrowing. At the time when the cramping pain occurs, Bill has ____________ compared to Bob. the same supply and the same demand more supply and the same demand the same supply and more demand more supply and more demand

more supply and the same demand Explanation As previously stated, the body has adaptations to increase supply of blood and oxygen during exercise, but these adaptations can only support a certain level of increase in supply. With his narrowed artery, Bob has increased resistance in the supply blood vessels and the blood and oxygen supply is limited relative to Bill when they are walking at a pace that brings on the cramping leg pain for Bob. Given that they are walking at the same pace, they both have the same demand. Bill has the supply needed to meet his muscles' oxygen demand, while Bob does not. In this case, the anaerobic metabolism triggered by exercise is best described as arising from normal/physiological range demand in the face of limited supply. This symptom (cramping pain in the legs during exercise as a result of arterial narrowing) is called claudication.

Which of the following is the best definition of steady state, as defined in this lesson? A system is in steady state when the amount of a given substance in the system is within a predefined normal range. neither increases nor decreases. is decreasing at a constant rate. is increasing at a constant rate.

neither increases nor decreases. Explanation A system is in steady state with respect to the levels of a substance when Input of the substance + Production of the substance = Elimination of the substance. If this condition is met, the total amount of the substance in the system remains at a constant level.

In the situation described above (short, high intensity exercise), what best describes the physiological balance of supply of oxygen and demand for oxygen in muscle tissue (assume normal vessels and normal muscle tissue)? limited supply and excess demand normal supply and excess demand limited supply and normal demand normal supply and normal demand

normal supply and excess demand Explanation The body has adaptations to increase supply of blood and oxygen during exercise, but these adaptations can only support a certain level of increase in supply. Above this threshold, the body turns to anaerobic metabolism. Therefore, anaerobic metabolism triggered by intense exercise is best described as arising from excess demand.

An infinitely flexible tube with an outside pressure of 100 mmHg is currently collapsed and therefore has no flow through it. The pressure inside the tube must become greater than ____________ mmHg in order to relieve the collapse and resume flow through the tube.

one-hundred or one hundred or 100 Explanation In order to resume flow through the tube, the transmural pressure must be positive. If the transmural pressure is 0 or negative, flow will not resume, as the volume of the tube will not expand and the tube will remain collapsed.

Suppose the gas packed in a champagne bottle is manipulated such that the total pressure of the gas is 760 mm Hg with a PCO2 of 720 mm Hg. PCO2 in the atmosphere is approximately 0. When the cork is pulled, does CO2 move: out of the bottle in to the bottle neither - the inside and outside of the bottle are already in equilibrium because the total pressure of the gas inside and outside is the same, so there is no movement of gas

out of the bottle Explanation: Pressure gradients for the individual gases (such as CO2) would cause the gases to move until equilibration between bottle and atmosphere occurs. Thus CO2 would move out of the bottle until the PCO2 reaches the PCO2 in the atmosphere (approximately 0).

The left-hand column below indicates a change to acid input, elimination, or production. Select the expected change in blood pH that best matches each scenario (assume that all other factors are held constant). Elimination increases Select an option pH increases pH decreases need more information Elimination decreases Select an option pH increases pH decreases need more information Production decreases Select an option pH increases pH decreases need more information Input increases Select

pH increases pH decreases pH increases pH decrease pH decreases need more information Explanation A system is in steady state with respect to the levels of a substance when Input of the substance + Production of the substance = Elimination of the substance. Since our system begins in steady state, we know that changing any one of the variables while holding the others constant will lead to a change in the levels of acid in the system. Any one of the following will increase the acid (decrease the pH) of the system: increasing production, increasing input, or decreasing elimination. Conversely, doing any one of the following will decrease the acid (increase the pH) of the system: decreasing input, decreasing production, or increasing elimination. In the two mixed scenarios, without further information about the magnitude of each change, we can know the overall result on the acid levels only if both changes push acid levels in the same direction. So in the increased production and decrea

As heart rate increases during exercise, time in diastole decreases. Assuming that the driving pressure for blood flow through the heart vessels does not increase, how is the increased oxygen demand of the left ventricle met during cardiovascular exercise? through a relatively equal contribution from oxygen extraction and decreasing vessel resistance primarily through increased diffusion of oxygen from the blood in the left ventricular chamber primarily through decreasing resistance via dilatio

primarily through decreasing resistance via dilation of heart vessels Explanation Remember that flow is expressed as volume per unit time. Blood flows through the heart vessels primarily during diastole, so blood flow to the myocardium would decrease as heart rate increases, were it not for a substantial vasodilation (and hence decreased resistance) of heart vessels during exercise. For the left ventricle, oxygen extraction is near maximum at baseline, so the primary mechanism for meeting the oxygen demand of the heart muscle is through increasing flow via vasodilation. This is in contrast to other tissues in the body, including skeletal muscle, where oxygen extraction is lower at baseline and can therefore play a great role when oxygen demand increases.

How would you describe the acid-base status of an individual during the anaerobic phase of exercise (i.e. when the individual has passed the anaerobic threshold)? primary metabolic acidosis primary respiratory acidosis primary metabolic alkalosis primary respiratory alkalosis

primary metabolic acidosis Explanation Increased production of lactic acid during the anaerobic phase of exercise results in a primary metabolic acidosis with respiratory compensation (increased minute ventilation expels the CO2 produced from buffering lactic acid with bicarbonate). The respiratory compensation does not fully correct the metabolic acidosis (i.e., pH will be below 7.40).

Oncotic force is a type of osmotic force due to having a differential concentration of ________ across the wall of a capillary.

protein or proteins Explanation: Since the vascular wall is minimally permeable to protein under normal conditions, there is more protein in the vascular compartment compared to the interstitial compartment. This higher concentration of protein in the vessels creates a force that acts to draw fluid into the vascular compartment.

A diabetic 21-year-old college student arrives in the hospital and is admitted to the medicine service. Laboratory tests indicate that he is markedly acidemic, with a blood pH of 7.12, likely due to diabetic ketoacidosis, a condition in which the body uses fat as a main fuel for energy generation due to a deficiency in insulin signaling. Knowing that the body has many chemoreceptors to sense H+ and CO2, you predict that his body will compensate to reduce the amount of acid in his blood. What wou

rapid respiratory rate deep breaths Explanation This patient's body will try to correct his acidemia almost immediately using the respiratory system. In an effort to reduce CO2 levels in the blood (and, consequently, the amount of carbonic acid), his ventilation will increase by increasing the respiratory rate and the tidal volume (amount of air moved in each breath). This deep and rapid style of breathing, known as Kussmaul respiration, is commonly seen in diabetic ketoacidosis.

As fluid moves along normal systemic capillaries, the oncotic pressure inside the capillary: stays roughly the same because essentially no protein moves in or out of the vessel and only a small amount of fluid leaves the vessel increases significantly because a large amount of fluid moves out of the vessel decreases significantly because a large amount of fluid moves out of the vessel decreases significantly because protein is lost from the vessel

stays roughly the same because essentially no protein moves in or out of the vessel and only a small amount of fluid leaves the vessel Explanation: The vascular wall is minimally permeable to protein. Since the hydrostatic pressure is greater inside the arterial end of the capillary than in the interstitium, water will move out of the capillary at the arterial end. The protein left behind in the capillary becomes slightly more concentrated as water exits to the interstitium but the change in protein concentration is insignificant, given that the percent of the total blood flow that ends up being filtered is very small (well under 1%).

Which area of the heart is particularly at risk for low levels of oxygen? Be as specific as possible.

subendocardium or subendocardial layer or sub endocardium or sub-endocardium or endocardium or endocardial layer or subendocardial or endocardial Explanation The subendocardium is most susceptible to low oxygen supply. This is due to two factors. First, the subendocardium is farthest from the blood supply; arterial blood must pass through penetrating vessels (vessels that go through the heart wall) that originate from epicardial coronary arteries. Because these vessels have some resistance, pressure decreases the farther downstream the blood moves. Therefore, the driving pressure for the subendocardial vessels is lower. Second, the subendocardial vessels experience higher external compression forces (than vessels in other layers of the heart muscle) due to their proximity to the cavity of the left ventricle and the high pressure of the blood within that chamber. This compression narrows the vessels, thereby increasing resistance and decreasing flow.

Assume a disease prevented the body tissues (as a whole) from changing oxygen extraction as cardiac output changed. What would happen to oxygen consumption as cardiac output falls? the body's oxygen consumption would stay the same the body's oxygen consumption would decrease the body's oxygen consumption would increase

the body's oxygen consumption would decrease Explanation The two primary factors determining oxygen consumption are blood flow (cardiac output for the body as a whole) and oxygen extraction. If blood flow decreases and oxygen extraction cannot increase, oxygen consumption of the body necessarily falls. Recall that the Fick equation describes oxygen consumption as the product of cardiac output and oxygen extraction; therefore, with decreasing cardiac output and constant oxygen extraction, the product (oxygen consumption) must decrease.

Standing up from a prone position (lying down) can cause a slight, but rapid decrease in blood pressure. In response to standing up after waking up in the morning, baroreceptor signaling will cause (select two answers) a decrease in sympathetic nervous system stimulation. the heart to contract more forcefully. blood vessels to constrict. heart rate to decrease.

the heart to contract more forcefully. blood vessels to constrict. Explanation Recall the "Ohm's law analogy" equation for blood pressure (P): MAP - CVP = Q x SVR. Cardiac output (Q) is determined by both stroke volume and heart rate, specifically Q = SV x HR, meaning we could rewrite the equation as ΔP = SV x HR x SVR. To increase the difference in pressure, i.e. increase the mean arterial pressure, you have to increase SV, HR, and/or SVR. Contracting the heart more forcefully increases SV and constricting blood vessels increases SVR. In response to a low blood pressure, HR would increase and there would be an increase in mediators of the sympathetic nervous system such as epinephrine.

Which part of the heart do you think is most at risk for low supply under conditions of high demand? both the subendocardial layer and the subepicardial layer are equally at risk the inner muscle layer of the heart (the subendocardial layer) the outer muscle layer of the heart (the subepicardial layer)

the inner muscle layer of the heart (the subendocardial layer) Explanation As you will learn in the next video, the subendocardial muscle layer of the heart is particularly susceptible to problems of an imbalance of supply and demand.

A patient with a history of smoking undergoes an exercise test on a treadmill. She can only run for 5 minutes before becoming short of breath. Her heart rate is 50% lower than predicted for an individual of her age after 5 minutes of exercise. What physiological system (respiratory or cardiovascular) is likely limiting her exercise capacity? the respiratory system the cardiovascular system

the respiratory system Explanation In healthy individuals, oxygen delivery (the primary function of the cardiovascular system) generally limits exercise capacity. However, this patient's heart rate was only 50% of her predicted maximum heart rate when she stopped exercising. This suggests that she has some cardiac reserve (healthy adults generally stop exercising at 85-95% of their predicted maximum heart rate). Given her history of smoking, lung disease may be limiting her exercise capacity.

Suppose we have two vessels with the same cross-sectional radius at baseline connected end to end (in series). Consider two scenarios (see image): in scenario A, vessel 1 is fully open and vessel 2 is constricted such that its diameter is 50% of its baseline maximum; in scenario B, vessel 1 has a 50% diameter fixed blockage (as might arise from atherosclerosis) and vessel 2 is fully open. Assuming the same driving pressure in both scenarios, blood flow through the vessels will be the same in s

the same in scenario A as in scenario B Explanation For resistances in series, the total resistance is the sum of the resistances. In this case, the total resistance of the system (the two vessels end to end) is the same. Based on the Ohm's Law analogy, flow through the vessels should be the same for both scenarios if driving pressure is the same. In fact, the heart uses this mechanism to compensate for fixed blockages by dilating vessels downstream of the blockage to maintain flow. This mechanism has its limits, based on the ability of vessels to dilate, as described elsewhere in this lesson.

When fluid flows through a tube, the orientation of the motion of the molecules relative to the axis of the tube determines how much the pressure of the fluid decreases as it travels along the tube. Laminar flow is present when the molecules in a fluid move in a direction parallel to the long axis of the tube. Under conditions of laminar flow, the change in pressure as the fluid flows through the tube is proportional to the flow (ΔP proportional to Flow). Under conditions of turbulent flow, on

tube 1-C; tube 2-B Explanation An infinitely flexible tube collapses when the transmural pressure becomes negative (when the pressure outside the tube exceeds the pressure inside the tube). Under conditions of turbulent flow, the pressure drops more steeply as fluid moves through the tube than under conditions of laminar flow; in other words, ΔP for turbulent flow > ΔP for laminar flow (since in turbulent flow, ΔP proportional to Flow2, while in laminar flow, ΔP proportional to Flow). ΔP is the difference between the pressure at point A and the pressure at point B in each tube. Since the pressure at point A is the same in both cases, a greater ΔP leads to a lower pressure at point B of tube 2 (the turbulent flow scenario) relative to tube 1 (the laminar flow scenario). This means that the pressure drops more drastically in tube 2 than in tube 1. Since we know that each tube collapses at either point B or C, we can say that tube 1 collapses at point C and tube 2 collapses at poin

In the cardiovascular system as a whole, the resistance of the systemic (non-pulmonary) circulation is called systemic vascular resistance (SVR) and the flow is called cardiac output (sometimes labeled as Q). ∆P in the Ohm's law analogy for the cardiovascular system is mean arterial pressure (MAP) minus ________ pressure.

venous or central venous or vein Explanation: Note that the "systemic" circuit (i.e., the part of the circulatory system taking oxygenated blood from the left ventricle and returning relatively deoxygenated blood to the right side of the heart) does not include the pulmonary circulation (i.e., the flow of deoxygenated blood from the right ventricle through the pulmonary circulation where it picks up oxygen in the lungs and then returns to the left side of the heart). The pulmonary vasculature is characterized by a separate measurement, called pulmonary vascular resistance (PVR).

In this lesson, we've mostly focused on oxygen and nitrogen partial pressures. What else (other than O2 and N2) may contribute to the total pressure in the alveoli? Enter a single answer (there may be more than one correct answer).

water or water vapor pressure or water pressure or H2O or carbon dioxide or CO2 Explanation: We've mostly focused previously on oxygen and nitrogen partial pressures because those are the primary components of inspired air. But once the air reaches the alveoli, it has been humidified (by the airways normally, or by the ventilator for this patient). In the alveoli, the capillaries offload the carbon dioxide produced by the body tissues, and thus CO2 also contributes to partial pressures in the alveoli.


Kaugnay na mga set ng pag-aaral

4.1 Solve one variable equations and inequalities

View Set

Pharm II Prep-U Ch.26 Opioid Agonists, Opioid Antagonists, and Antimigraine Agents

View Set

CIS 150 Midterm (online quiz questions)

View Set

NUR 1511 Mental Health Chapter 18: Personality Disorders

View Set

Random extra study questions for exam 2

View Set

Psychology - Hensley FINAL ( chapter 12 )

View Set